You are on page 1of 483
1. NTA UGC NET 2020 (Morning Shift, Paper-II) 1, The lives of which of the following writers has been the subject matter of novels by Anthony Burgess? (A) Milton (B) Marlowe (Shelley (D) Keats Choose the Correct answer from the options given below: (1) AandB only (2)AandD only (3) Band C only (4)BandD only Answer: (4)[Anthony Burgess (1917-1993) was one of the most prolific literary figures of the 20° century, producing a large number of novels, plays, biographies, screenplays, criticism, and articles. He wrote novels based on the lives of poets. His novel Nothing Like the Sun is based on Shakespeare and A Dead Man in Deptford isabout Christopher Marlowe. He also wrote a novel about the last months of John Keats called Abbadbba (1977). ‘Abba’ meaning ‘father’. The words ‘Abba Abba’ appear on Burgess’ gravestone also. The verse novel Byrne was published posthumously.] 2. Match List I with List It List List It ‘Terms ‘Theorist A Arche-eeriture 1. Julia Kristeva B. Cyborg, TI, Donna Haraway C.Genotext Ill. Fredrich Schleiermacher D. Hermeneutic Circle IV. Jacques Derrida Choose the Correct answer from the options given below: ()A-IV,B-, C1, D-l Q)A-II,B-1,C-,DIV (@)A-II,B-H, C-IV,D4 (4) A-IV,B-1,C-, D-IL Answer: (1)[Arche-writing is a term used by Jacques Derrida in his attempt to re-orient the relationship between speech and writing, A cyborg is any self-organizing system which combines organic and mechanical parts. The word was coined by Manfied E. Clynes and Nathan S, Kline in their 1960 Astronautics article ‘Cyborgs and Space’. However, critical theory did not explore the implications of the cyborg until the American socialist-feminist Donna Harawaywrote her seminal ‘Cyborg Manifesto’ (1985). Harawayreinscribed the cyborg as a political and theoretical idea which could disrupt conventional binary oppositions, such as human/animal and organism/ machine. Because the cyborg is a hybrid or mixture, it suggests an alternative to unifying, homogeneous concepts, such as ‘Woman’, For Haraway, the cyborg is a liberating myth which represents ‘transgressed boundaries, potent fusions, and dangerous possibilities which progressive people might explore as one part of needed political work? According to Julia Kristevasemiology is based on two concepts -genotext and phenotext. The genotext is the body of the bio-physiological process constrained by the social code, and it is not reducible to the language 2 BRUSHUPYOURENGLISHLITERATURE system, The genotext exists within the phenotext, which is the perceivable signifying system, Hermeneuticsis the art or technique of interpretation. The development of a defined area of modern theory termed hermeneutics out of the practice of scriptural interpretation was predominantly the work of Friedrich ‘Schleiermacher (1768-1834), Wilhelm Dilthey (1833-1911), Martin Heidegger (1889-1976), and Hans-Georg Gadamer (1900-). In recent theory hermeneutics has often been regarded as an approach to Texts that is no longer methodologically defensible. Post-structuralism sees itself as renouncing hermeneutics’ metaphysical goal of finding the text’s original meaning.] 3. Match List I with List 11 ist List ‘Author. ‘Text ‘AMichel de Certeau I. Distinction B. John Fiske II Reading of the Romance C. Pierre Bourdieu II Understanding Popular Culture D.Janice Radway IV. The Practice of Everyday Life n below: Choose the Correct answer from the options ()A-IV,B-L, CI, D-II (Q)A-IL,B-IV,C-, Dl @)A-IV,B-IN, CA, DT (4)A-I, B-IIl, CI, D-IV Answer: (2)[Pierre Bourdieu’sDistinction: A Social Critique of the Judgement of Taste (1984) explores the social and economic production of a distinction between high art and popular culture. Originally published in 1984, Reading the Romance challenges popular myths about why romantic fiction, one of publishing’s most lucrative categories, captivates millions of women readers. Among those who have disparaged romance readings are feminists, literary crities, and theorists of mass culture. John Fiske’s Understanding Popular Culture (1989) took a groundbreaking approach to studying such cultural artifacts as jeans, shopping malls, tabloid newspapers, and TV game shows, which remains relevant today. Fiske differentiates between mass culture ~ the cultural “products” put out by an industrialized, capitalist society — and popular culture ~ the ways in which people use, abuse, and subvert these products to create their own meanings and messages. Michel de Certeau’s The Practice of Everyday Life (1980) examines the ways in which people individualise mass culture, altering things, from utilitarian objects to street plans to rituals, laws and language, in order to make them their own,] 4. Arrange the following plays in their chronological order: A The Country Wife B. Cymbeline C. The Spanish Tragedy D. The Rivals Choose the Correct answer from the options given below: ()B,A,CD )B,C,D,A @CBAD CAB, Answer: (3) [The Spanish Tragedy is a play by Thomas Kyd that was first performed in 1587. Cymbeline is a comedy by William Shakespeare. Itis one of his later plays, written in 1608-10 and published in the First Folio of 1623. The Country-Wife is a comedy of manners in five acts byWilliam Wycherley, performed and published in 1675. The Rivals is a comedy by R. B. Sheridan, produced and published in 1775.] 5. Which of these statements describe correctly the basic assumption of Structuralism? A. Structuralism is concerned with signs and signification B.A structuralist theory considers only verbal conventions and codes. C. Structuralism began in the works of Jacques Derrida and influenced the 20 th century literary criticism. D Structuralism challenges the long- standing belief that literature reflects a given reality. E. All signs are arbitrary but without them we cannot comprehend reality. ‘Choose the Correct answer from the options given belo (1)A, Cand E only (2)A, Dand E only (3)A, Band C only (4)A, Band E only Answer: (2)[Structuralism isa method of inquiry whichproceeds from the premise that cultural activity can be approached and analyzed objectively as a science.Structuralism is usually considered to have begun with the linguistics of Ferdinand de Saussure. Saussure called for a science of signification, Semiotics, of which linguistics was to be the privileged example. The linguistic Sign is structured as a relation between the signified (the referent) and the signifier (the lexical unit), Signs are structured in relation to each other in accordance with the principle of differential relations, but are also organized in a further relation. This relation is the one which constitutes Binary oppositions, relations between opposites which are charged with important meanings in human Culture.] 6, Who among the following wrote Mazeppa, a long narrative poem about seventeenth century military leader of Ukraine? (1) William Cowper (2)Lord Byron ()P.B. Shelley (@)S.T.Coleridge Answer: (2)[Mazeppa is a narrative poem written by Lord Byron in 1819. It is based on a popular legend about the early life of Ivan Mazepa (1639-1709), who later became the military leader of Ukraine.] 7. Which of the following aspects are to be scrupulously followed to avoid the trap of plagiarism? A Subjectivity B. Acknowledgement C. Citation D. Interpretation Choose the most appropriate answer from the options given below: (A and Bonly (2)A and C only (3)C and D only (4)B and C only Answer: (4)[Plagiarism is presenting someone else’s work or ideas as your own, with or without their consent, by incorporating it into your work without full acknowledgement. It may be intentional or reckless, or unintentional. Under the regulations for examinations, intentional or reckless plagiarism is a disciplinary offence. To avoid plagiarism one should Keep track of the sources he consults; Paraphrase or quote from one’s sources; and credit the original author in an in-text citation and reference list.] 8. Which of the following journals publishes articles related to critical theory exclusively? (1) Salmagundi (2) Diacrities 3)Callaloo (4) Gand Street Answer: (2) [Founded in 1971, Diacritics publishes original work in and around critical theory. Diacritics offers a forum for thinking about contradictions without resolutions; for following threads of 4 BRUSHUPYOURENGLISH LITERATURE contemporary criticism without embracing any particular school of thought. For Diacritics, eclecticism in the humanities means nurturing work that is trans-historical, creative, and rigorous.] 9. Which of the following poems are by Robert Browning? A.“Locksley Hall” B. “The Pied Piper of Hamelin” C. “The Lady of Shallot” D. “Two in Campagna” ‘Choose the most appropriate answer from the option: (1) Aand D only (2) Band C only (3) Aand C only (4) Band D only Answer: (4) ["The Pied Piper of Hamelin” and “Two in Campagna” are poems by Robert Browning. “Locksley Hall” and “The Lady of Shallot” are poems by Alfred Tennyson.] 10. Who among the following is a set of Metaphysical poets? (1) John Dryden, George Herbert, and Alexander Pope (2) Henry Vaughan, John Dryden and John Donne (3) John Donne, Henry Vaughan and Andrew Marvell (4) Samuel Johnson, T. S. Eliot and Herbert Grierson Answer: (3) [Metaphysical poet, any of the poets in 17%century England who inclined to the personal and intellectual complexity and concentration that is displayed in the poetry of John Donne, the chief of the Metaphysicals. Others include Henry Vaughan, Andrew Marvell, John Cleveland, and Abraham Cowley as well as, toa lesser extent, George Herbert and Richard Crashaw.] 11, Which two of the following are the titles of the sections in Thomas De Quincey’sThe English Mail Coach? ‘A. The Glory of the Mobility B. The Vision of Sudden Death C. The Glory of Motion D, The Vision of Unexpected Truth ‘Choose the most appropriate answer from the options given below: (1) Aand B only (2)Aand D only (3) Band C only (4)BandD only Answer: (3)[One of his most magnificent worksThe English Mail-Coach (1849) is a “three-part masterpiece” essay by Thomas De Quincey. The first Part is “The Glory of Motion”; Second partis “The Vision of Sudden Death” and third part is “Dream Fugue, Founded on the Preceding Theme of Sudden Death” ] 12. Which of these statements defines the scope of semiotics (1) Semiotics studies the sound system of a language (2) Semiotics is a study of sign system (3) Semiotics studies human sign system only (4) Semiotics is a study of non- human sign system only Answer: (2) [Semiotics isthe science of Signs. Umberto Eco has wittily defined semiotics as the study of everything that can be taken as a sign. “A sign is everything which can be taken as significantly substituting for something else. This something else does not necessarily have to exist or to actually be somewhere at the moment when sign stands in for it. Thus semiotics is in principle the discipline studying everything which can be used in order to lie.] NTA UGC NET 2020 (Morning Shift, Paper-II) 5 13. Arrange the following in the chronological order of publication: A. Crome Yellow B. Sons and Lovers C. Mrs. Dalloway D. A Portrait of the Artist as a Young Man ‘Choose the correct answer from the option: ()B,A.C,D (Q)A,B,D¥ @)ACBD (4)B,D,A,C Answer: (4) [D. H. Lawrence’s autobiographical novel Sons and Lovers was published in1913. James Joyce's first novel A Portrait of the Artist as a Young Man was published in 1916. Aldous Huxley’s first novel Crome Yellowwas published in 1921. Virginia Woolf’s novel Mrs. Dalloway was first published in 1925.] 14. Which of the following works does Walter Pater regard as examples of “great- art” in his essay “Style”? iven below: A.lliad B. The Divine Comedy C. Les Miserable D. Faust ‘Choose the most appropriate answer from the options given below: (1) Aand B only (2)AandD only (3) Band C only (4)BandD only Answer: (3) [Walter Pater was an English critic, essayist, and humanist whose advocacy of “art for art’s sake” became a cardinal doctrine of the movement known as Aestheticism. Discussing his principles of composition in the 1888 essay “Style” Pater writes, “Good art, but not necessarily great art; the distinction between great art and good art depending immediately, as regards literature at all events, not on its form, but on the matter. Thackeray's Esmond, surely, is greater art than Vanity Fair, by the greater dignity of its interests. It is on the quality of the matter it informs or controls, its compass, its variety, its alliance to great ends, or the depth of the note of revolt, or the largeness of hope in it, that the greatness of literary art depends, as The Divine Comedy, Paradise Lost, Les Miserables, The English Bible, are great art.” ] 15, Mateh List I with List-II List-1 List-Il Lines Poems A Monuments of unaging intellect, I. Leda and Swan Tn the foul- rag and bone shape of the heart | I. Adam’s Curse So mastered by the brute blood of the air__| IIL. Sailing to Baizaintium ‘AS weary hearted asthat hollow moon TV. The Circus Animal's desertion ‘Choose the correct answer from the options given below: (1) A-IIL, B-IV,C-11,D-1 Q)A-H, B-I, C-IV, D-Il (3).A-III, B-IV, C-I, D-T (4) .A- II, B-1,C-IV, D-IL Answer: (2) [The line “A Monuments of unaging intellect” appears in W. B. Yeats’s poem “Sailing to Baizaintium”, The line “In the foul- rag and bone shape of the heart” occurs in the poem “The Circus Animal’s desertion”. The line “So mastered by the brute blood of the air” is taken from W. B. Yeats’s poem “Leda and Swan”. The line “As weary hearted asthat hollow moon” is occurred from the poem “Adam’s Curse”.] 16. In which of the Bog poems does Seamus Heaney speak about the “ perishable treasure” of a body 6 BRUSHUPYOURENGLISH LITERATURE “murdered, forgotten, nameless, terrible”? (1) “Bog Queen” (2) Grauballe Man” (3) “Punishment” (4) “Strange Fruit” Answer: (4)[In his collection North (1975) Seamus Heaney included four bog body poems, “Bog Queen”, “The Grauballe Man”, “Punishment”, and “Strange Fruit”. In these poems, Heaney draws connections between the past and present. Unlike the other poems, the descriptions in “Strange Fruit” do not evoke the body's past life and describe it as a bog body. Heaney describes the body as it has been preserved and how this body gradually moves into the “Murdered, forgotten, nameless, terrible.” The body tales a new ethos as something that forgets whatever past life of the body. It is a testament to the terrible legacy created because of war] 17. Which of the following two books are explorations of artof the novel by novelists? A. The Brief Compass B. The Naive and the Sentimental Novelist C. The Visionary Company D. Testament Betrayed ‘Choose the most appropriate answer from the options given below: (1) Aand Bonly (2)AandC only (3) Band C only (4) Band D only Answer: (4) [OthanPamuk’s7he Naive and the Sentimental Novelist(2010) is an inspired, thoughtful, and deeply personal book about reading and writing novels. In this fascinating set of essays, based on the talks he delivered at Harvard University as part of the distinguished Norton Lecture series, Pamuk presents a comprehensive and provocative theory of the novel and the experience of reading, Milan Kundera’s Zestaments Betrayed (1993) is abrilliant and thought-provoking essay written like a novel: the same characters appear and reappear throughout the nine parts of the book, as do the principal themes that preoccupy the author. ] 18. Who among the following coined the dictum, “the medium is the message”? (1) Reymond Williams (2) Erving Goffman (3) Marshall Mcluhan (4) John Fiske Answer: (3)[“The medium is the message” is a phrase coined by Marshall McLuhan and introduced in his Understanding Media: The Extensions of Man, published in 1964. The phrase sums up a much deeper communication theory, which is that the medium through which we choose to communicate holds as much, if not more, value than the message itself. ] 19, Which two terms among the following are specially linked to the work of Pierre Bourdieu? A. Habitus B. Consciousness C. Desire D. Distinction Choose the most appropriate answer from the options given below: (2) Aand C only (2) Aand D only (3) Band D only (4) CandDonly Answer: (2)[Pierre Bourdieu (1930-2002) was a Frenchsociologist, philosopher and anthropologist whose work explored a wide range of subjects including art, education, language, the mass media and globalization. In his best-known book, Distinction: A Social Critique of theJudgement of Taste (1984), he uses these categories to explore the social and economic production of a distinction between high art and popular culture, arguing that one is not intrinsically better than the other, but that such a distinction is produced by the conflict between social NTA UGC NET 2020 (Moming Shift, Paper-II)_ 7 fields. The term habitus often associated with Pierre Bourdieu referring to the mental or cognitive structures via which people interact with the larger social worlds in which they live. Bourdieu defines habitus as systems of internalised or embodied social structures that can change over time as individuals acquire new or different associations and experiences; such sets of relations and attitudes exist within what Bordieu terms a field.] 20. Which of the two characters/ speakers among the following exhibit the studious abstraction of scholars? A. Shylock B. Hamlet C. IlPenseroso D. Mosca Choose the most appropriate answer from the options given below: (A and D only (2)B and C only (3)Cand D only (4)A and C only Answer: (2)[A companion piece to L Allegro, Milton's I! Penseroso (1631) is written inthymed octosyllabics. The title means ‘the contemplative man’. The poem is an invocation to the goddess Melancholy, bidding her bring Peace, Quiet, Leisure, and Contemplation. It describes the pleasures of the studious, meditative life, of tragedy, epic poetry, and music. It had a considerable influence on the meditative graveyard poems of the 18* century. Shakespeare's Hamlet is considered as ‘a Mona Lisa in Literature’. The character of hamlet is very complex and had been a subject of discussion among the scholars] 2 List-1 List-It ‘Terms ‘Theorist ‘A Supe-reader T. Michel Foucault B. Biopower TL Mikhail Bakhtin C. Bricolage TH. Michael Riffatere D. Chronotope TV. Claude Levi Strauss ‘Choose the correct answer from the options given below: (1) A- IIL, B-II, C-IV,D-1 (2)A-IM, B-I, C-IV, D-II Q)ACIV,B-1,C-II,D-11 (4) AIL, B-I, C-IV, D-II Answer: (2)[Michael Riffaterre coined the term super reader. The super reader is like a palimpsest of available textual commentary on the text which may include the author’s statements or corrections, translations, and dictionaries. The super reader is another system, a system of inter-textuality. All the responses of the text have recorded by super reader. The super reader isa collective term for a variety of readers of different competence that Riffaterre uses, “to discover a density of meaning potential encoded in the text” According to Foucault, power is the sovereign’s right to rule. Biopower controls vital processes. It is a health approach suitable for a process of standardization: “biopower keeps us alive”. Therefore, Foucault falls in line with Lukacs with his notion of class consciousness, although he wavers from one end of the spectrum to the other. Bricolageis a term used by Claude Levi-Straussto describe the mentality of the non-industrialized subject, or ‘primitive man’. In The Savage Mind (1966), Levi-Strauss compares primitive man to the bricoleur, Chronotopeis aterm coined by Mikhail Bakhtinin the 1930s to describe the way in which time and space are 8 BRUSHUPYOUR ENGLISH LITERATURE represented and connected in literature. In his essay “Forms of Time and the Chronotope of the Novel’, Bakhtin offers a history of the novel, which aims to show that different novels are structured according to different ideas of time and space.] 22, Who among the following believe that rhyme is not an integral part of poetry? A. William Wordsworth B. Horace C. Samuel Daniel D. Philip Sidney ‘Choose the most appropriate answer from the options given below: (1) Aand C only (2)BandD only (3)Aand D only (4)D and C only Answer: (2) [Horace believed that Poetry is not mare “rhyming and versifying”. He believed that great poetry must be both pleasure giving and morally improving. Sidney also points out that rhyme is not the essence of poetry but it is desirable to use it for the following reasons: 1. itis a polish to speech. 2. Scaliger had defended its use. 3. it regulates verbal harmony. 4. it adds to words the sensuousness and emotional quality of music. 5. itis an aid to memory.] 23. LalaKashi Ram is a character in (1) Arun Joshi’s The Apprentice (2) ChamanNahal’s Azadi (3) Anita Desai’s Fire on the Mountain (4) Kamala Markandaya’s A Handfidl of Rice Answer: (2) [Chaman Nahal’s Azadi (1975) is a modem classic which portrays the realistic historical documentation of the atrocious confrontations caused by the partition through literary perspective. As ‘ChamanNahal himself was a refugee, he writes with incredible ingress and realism. Therefore he has written his, ‘own experience through the character of LalaKashiram and his son Arun. In Azadi ChamanNahal, through his protagonist, LalaKanshi Ram also conveys his idea of partition and the British rule. LalaKanshi Ram has dual feelings towards the British. He admires them for their qualities, but criticizes them for their faults.] 24, Which among the following novels include a questionnaire for the readers such as “do yo story so far? Yes () No()” ? (1) Mantissa by john Fowles (2) Waterland by Graham Swift (3) Snow White by Donald Barthelme (4) Ifon a Winter's Night a Traveller by Italo Calvino Answer: (3)[Snow White (1967) is a post-modernist novel by author Donald. It inverts the fairy tale of the same name by highlighting the form by discussing the different expectations and compromises the characters make to survive in their world, At the end of part one is a questionnaire is given. There are, in total 15 questions. Some famous of theme are: 1. Do you like the story s0 far? Yes ( ) No ()2. Does Snow White resemble the Snow White you remember? Yes () No ()3. Have you understood, in reading to this point, that Paul is the prince-figure? Yes () No ( )4. That Jane is the wicked stepmother-figure? Yes () No ()-] 25, Macaulay's Minutes of 1835 sought to: A. promotes European literature and science among the natives. B, Impart knowledge of English literature and science through translated texts. C. Encourage branches of native learning by more useful studies. D, Stop expenditure on the publication of oriental works and spend funds only on English education Choose the most appropriate answer from the options given below: ike the NTA UGC NET 2020 (Morning Shift, Paper-I)_ 9 (1)AandD only (2)BandD only (3) Aand C only (4) Band C only Answer: (1) [Macaulay's Minute of 1835 is a great historical document in Indian Education. It brought a ‘great effect on the educational system and policy in shaping the modem Indian Education System. He wanted to create a pool of Indians who would be able to serve British interests and be loyal to them. This class would be “Indian in blood and colour, but English in tastes, in opinions, in morals and in intellect.” Macaulay wanted the government to spend money only on imparting western education and not on oriental education. He advocated the shutting down of all colleges where only eastern philosophy and subjects were taught.) 26. Which is the subject of Ivan’s controversial essay in Brother Karamazov? (1)Transubstantiation (2) The Evils of clergy (3) The Eucharist (4) Ecclesiastical Courts Answer: (4) [Ivan is a fictional character in Fyodor Dostoevsky’s novel The Brothers Karamazov. He is an atheist and has a deep compassion for the sufferings and tribulations of earthly man. He refrains from questioning the existence of God but refuses to accept this world as being God’s world. From his questionings Ivan has developed a long prose poem entitled “The Grand Inquisitor” in which he envisions Christ returning to earth. He is again threatened with death, but this time He is indicted by the church. Christ’s second death is demanded because the cardinal explains that mankind is too debased to accept the ideas advocated by Christ. The church, consequently, has taken away the freedom that Christ promised man, and for man’s good it has enslaved him. In this poem, Ivan reveals the depths of his compassion for mankind, creatures who he feels do not have the strength to follow the strenuous demands made by Christ.] 27. Which of the following poems by Philip Larkin deals with the trauma of a rape victim who says “Even so distant, I can taste the grief”? (1) “Deceptions” (2)**Faith Healing” (3) “Sad Steps” (4)“Wild Oats” Answer: (1) [“Deceptions”is a poem by Philip Larkin, It tells of the aftermath of a young woman’s rape and the way both rapist and victim were changed.A young woman who was raped tells of her distress after the attack ‘was over. In the second stanza the poet thought that by attacking this woman that his desire would be sated. He was deceived, as the title suggests. His deception is said to be worse than that suffered by the woman. This implies that the woman was tricked into the situation in which she was attacked.] 28, Which of the following dramatists are associated with the Epic Theatre? A. Fernando Arrabal B, Bertolt Brecht C.AmoltBronen D. James Saunders ‘Choose the correct answer from the options given below: ()A and B only (2)BandC only (@)A and D only (4)BandD only Answer: (2)[Epic theatre is a theory of theatrical presentation developedby Bertolt Brecht and outlined in his 1930 essay “The Modern Theater Is the Epic Theater.” Brecht created a type of drama that the audience could view with complete detachment. He usedwhat he termed “alienation effects” to create anemotional distance between the audience and theaction on stage. Among these effects are: short,self-contained scenes that keep the play from building to a cathartic climax; songs that comment on the action; and techniques of acting that prevent the actor from developing an emotional identity with his role. Besides the plays of Bertolt Brecht, such as A Man 10 BRUSHUPYOURENGLISH LITERATURE, Isa Man in 1926 and The Three penny Operain 1928, other plays that use epic theater conventions include those of Georg Buchner, Frank Wedekind, Erwin Piscator, AmoltBronenand Leopold Jessner. 29. Which two rivers are mentioned by Andrew Marvell at the beginning of “To His Coy Mistress”? A. The Ganges B. Thames C. Humber D. Jhelum ‘Choose the correct answer from the options given below: (1)Aand D only (2)A and B only @)AandC only (4)B and C only Answer: (3)[) [“To His Coy Mistress” is a carpe diem poem by Andrew Marvell. It urges a young woman to enjoy the pleasures of life before death claims her. Indeed, the poem is an attempt to seduce the titular “coy mistress.” In the process, however, the speaker dwells with grotesque intensity on death itself. Death seems to take over the poem, displacing the speaker's erotic energy and filling the poem with dread. The lover says “If we had all the time in the world, your prudishness wouldn’t be a problem. We would sit together and decide how to spend the day. You would walk by the river Ganges in India and find rubies; I would walk by the river Humber in England and write my poems. I would love you from the very start of time, even before the Biblical Flood; you could refuse to consummate our relationship all the way until the apocalypse.”] 30. Which two texts among the following are linked to literary feminism? A.A Small Place B. The Yellow Wallpaper C.Emma D.A Room of Once Own ‘Choose the correct answer from the options given below: (DA and D only (2)Cand D only (G)B and D only (4A and C only Answer: (3) [Literary feminism seeks on the one hand to investigate and analyze the differing representations of women and men in literary texts and, on the other hand, to rethink literary history by exploring an often marginalized tradition of women’s writing, Feminist criticism is concerned to question and challenge conventional notions of masculinity and femininity; to explore ways in which such conventions are inscribed in a largely patriarchal canon; and to consider the extent to which writing, language and even literary form itself are themselves bound up with issues of gender difference. Charlotte Perkins Gilman’s short story “The Yellow Wallpaper” (1928) is regarded as an important early work of American feminist literature for its illustration of the attitudes towards ‘mental and physical health of women in the 19* century. Virginia Woolf’s essay A Room of Ones Own (1929) addresses the status of women, and women artists and asserts that a woman must have money and a room of her own if she is to write. 31. In Anxiety of Influence which of the following definitions is given by Harold Bloom to exp! ‘clinamen”? (1) Poetic hyperbole (2) Poetic Misprision (3) Poetic Sublime (4) Poetic supplement Answer: (2)[In his book The Anxiety of Influence: A Theory of Poetry ( 1973) Harold Bloom introduces his six revisionary ratios which he consistently applies in this book as well as his suecessor volume titled A Map of Misreading. These are- Clinamen, Tessera, Kenosis, Daemonization, Askesis, Apophrades.. Bloom took the the term, NTA UGC NET 2020 (Morning Shift, Paper-II) 11 word Clinamen from Lucretius, who refers to swerves of atoms that make change possible, He defines this as “poetic misreading or misprision proper”. The poet makes a swerve away from the precursor in the form of a “corrective movement”. This swerve suggests that the precursor “went accurately up to a certain point”, but should have swerved in the direction that the new poem moves.} 32. Given below are two statements: one is labeled as Assertion A and the other is labeled as Reason R Assertion A: Research methods are a range of tools that are used for different types of inquiry. Reason R: The tools used in research are products of situations in which they are applied. In the light of the above statements, choose the correct answer from the options given below: (1) Both A and R true, and R is correct explanation of A (2) Both A and R true, and R is not correct explanation of A (3) A is true Ris false (4) Ais false but R is true Answer: (2) [Both statements are correct.] 33. Which one of the following statements is appropriately true of Harold Pinter’s plays? (1) Menace is in the air and it leads to bloody violence (2) Menace is in the air and itis realized through the female characters (3) Menace is in the air, but itis not pinned down, or explained. (4) Menace is in the air and anarchy follows in a systematic manner. Answer: (3) [Many of Harold Pinter’s plays are considered “comedy of Menace”. A comedy of menace is a play in which the laughter of the audience in some or all situations is immediately followed by a feeling of some impending disaster. The audience is made aware of some menace in the very midst of its laughter. The menace is produced throughout the play from potential or actual violence or from an underline sense of violence throughout the play. The actual cause of menace is difficult to define: it may be because, the audience feels an uncertainty and insecurity throughout the play Harold Pinter’s The Birthday Party is a comedy of menace. The play is actually the mingling of comedy with a perception of danger that pervade the whole play. Stanley, the central protagonist always finds his life beset with danger. When two persons called Mr. Goldberg and Mr. McCann come to stay in the same boarding house for a couple of nights. Their appearance fills Stanley's mind with unexplained fear and tension. The feeling of menace is reinforced when Stanley scares Meg by saying that some people would be coming that very day in a van, They would bring a wheelbarrow with them to take someone away. Eventually no one comes but Mr. Goldberg and Mr. McCann take stanley with them. ] 34, Who among the following linguists proposed the terms, ‘competence’ and ‘performance"? (1) Noah Webster (2) Steven Pinter (3) Roman Jakobson (4) Noam Chomsky Answer: (4) [Terms ‘competence’ and ‘performance’ are introduced into linguistics by Noam Chomsky. Competence refers to the knowledge that a native speaker has of a language. The term was contrasted with performance, the actual use of language in concrete situations. The distinction between competence and performance has been bitterly criticized, but the criticisms are groundless, since the distinction underlies virtually all work in linguistics, Chomskyan or otherwise] 35. “Hari wrote a poem on the mountain”, Which two of the following are admissible statements about the above sentence? ‘A. The sentence is an example of lexical ambiguity. B. The sentence is an example of structural ambiguity. C. The sentence involves two deep structures. D. The sentence involves two surface structures ‘Choose the correct answer from the options given below: (I)A and B only (2) Band C only 12 BRUSHUPYOURENGLISHLITERATURE (3) Band D only (4)CandD only Answer: (2) [The sentence is an example of structural ambiguity. Structural or syntactic ambiguity is the potential of multiple interpretations for a piece of written or spoken language because of the way words or phrases are organized. In the sentence we are not sure that Hari has written a poem about the mountain or by sitting on the mountain, The terms deep structure and surface structure were introduced by Noam Chomsky as a part of his work on transformational grammar. As per Chomsky deep structure refers to concepts, thoughts, ideas & feelings whereas surface structure refers to the words / language we use to represent the deep structure.Deep structure is the underlying syntactic structure—or level—of a sentence. It is an abstract representation that identifies the ways a sentence can be analyzed and interpreted. The above sentence can be analyzes by two ways thus it involves two deep structures.) 36. Match List-I with List- It List-1 List ‘Author ‘Work A. John Keats I. Alastor B, William Wordsworth IL. Song of Experience C.P.B. Shelley TIL Lamia D. William Blake IV. The Excursion Choose the correct answer from the options given below: (1)A-IIL, B-I, C-IV, D-II Q)A-IIB-IV,C-1,D-I1 (3)A-1,B-IV, C-IIl, D-IL (@)A-IV,B-II, C-1, D-II Answer: (2)[Alastor, ot The Spirit of Solitude is a poem by P. B. Shelley and first published in 1816.Songs of Experience (1789) s a collection of illustrated poems by William Blake. “Lamia” is a narrative poem written John Keats and published in July 1820. The Excursion is a long poem by William Wordsworth and was first published in 1814) 37. Arrange the following in the chronological order of publication: A. Advancement of Learning B. The Origin of Species C. On Heroes and Hero Worship D. The Lives of Poets Choose the correct answer from the options given below: ()D,A,C,B (2)D, A,B,C @)A,D.C.B (4)A,D.B,C Answer: (3) [Francis Bacon’s The Advancement of Learning was published in 1605,Samuel Johnson's Lives of the Most Eminent English Poets was published in 1779.Thomas Carlyle’ On Heroes, Hero-Worship was published in 1841 Charles Darwin's On the Origin of Species was published on 24 November 1859.] NTA UGC NET 2020 (Moming Shift, Paper-II) 13 38 Match List-I with List- I List-I List IL Linguist Concept ‘APaul Grice 1. Language Death B. Edward Sapir TI. Linguistic Signs (C Ferdinand De Saussure IL Linguistic Relativity D.NaneyDorin TV. Cooperative Principle Choose the correct answer from the options given below: ())A-I, B-III, C-I,D-IV (2)A-IVB-III, C-ILD-I @)A-IIL,B-IV,C-LD-11 (4) A-IIB-IV,C-IL,D-I Answer: (2) [The linguist Herbert Paul Grice developed a mode of interaction for successful communication called the Cooperative Principle and its maxims based on ordinary language philosophy. Linguistic relativity was first developed by Edward Sapir and Benjamin Lee Whorf, and is known as the Sapir-Whorf hypothesis or the principle of linguistic relativity (1956). It describes the idea that language influences the perceptions and thoughts of people, thus affecting their behavior. Linguistic signs are bilateral; consist of a signifier, made up of speech sounds and a signified, created by the sign’s sense content. The concept of Linguistic signs was given by Ferdinand De Saussure. A great contribution to the field of linguistics Nancy Dorin’sLanguage Death: The Life Cycle of a Scottish Gaelic Dialect describes types of changes in languages due to time.] 39, Arrange the following terms in the chronological order of emergence: A. Heresy of Paraphrase B, Stream of Consciousness C. Practical Criticism D. Defamiliarization ‘Choose the correct answer from the options given below: ()D,B.CA (2)B,D,a,C @)B,D,CA (4)D,C,B,A, Answer: (3)[The term Stream of consciousness was first used by the psychologist William James in The Principles of Psychology (1890).The Russian Formalists’ concept of “Defamiliarization”, proposed by Viktor Shklovsky in his Artas Technique (1817) refers to the literary device whereby language is used in such a way that ordinary and familiar objects are made to look different. The term ‘practical criticism’ is applied to an academic procedure devised by the critic I. A. Richards in his book Practical Criticism (1929)The term “heresy of paraphrase” was coined by Cleanth Brooks in his book The Well Wrought Urn (1949). The heresy is that of assuming that the meaning of a work of art (particularly of poetry) can be paraphrased. } 40. Which according to Thomas Hobbes is the only ‘science’ God has bestowed on mankind that informs the structure of his monumental work Leviathon? (1) Astronomy (2) Architecture (3) Occult Sciences (4) Geometry Answer: (4)[The 17° Century English philosopher Thomas Hobbes is widely regarded as one of a handful of truly great political philosophers, whose masterwork Leviathan and his “social contract theory” are 14 BRUSHUPYOURENGLISHLITERATURE stillremembered. Hobbes believed that the only ‘science’ God has bestowed on mankind is geometry.His philosophical method in Leviathan is modeled after a geometric proof, founded upon first principles and established definitions, and in which each step of argument makes conclusions based upon the previous step.] 41. Poetry according to Sir Philip Sidney is of three kinds. They are: (1) Religious, dramatic, romantic (2)classical, romantic, neo-classical 3)philosophical, imaginative, narrative (4) Religious, philosophical, imaginative Answer: (4)[In his treatise An Apologie for Poetrie Philip Sidney descries three kinds of poetry: (a) religious poetry, (b) philosophical poetry, and (c) imaginative poetry. He calls special attention to the third class of poets, for “these be they that, as the first and most noble sort may justly be termed vates.’ They ‘most properly do imitate to teach and delight, and to imitate borrow nothing of what is, has been, or shall be, but range, only with leamed discretion, into the divine consideration of what may be, and should be.”] 42. Who among the following theorists particularly emphasized the social and historical dimensions of text’s reception? (1) Wolfgang Iser (2) Stanley Fish (3) Hans Robert Jauss (4) Pierre Bourdieu Answer: (3)[Hans Robert Jausswas associated with the Konstance School reception aesthetics. Jauss’s ‘main concern lies in how literary texts have been received at different timesor epochs. His approach consequently incorporates a mode of historical thought: that is, awareness that a text does not exist as an independent or a- temporal entity but requires realization in a specifichistorical context, Since a text does notexist outside history, it can be thought of asalways historically situated. Jauss himselfargues that a text is situated within a horizonof. expectations which is constituted bya specific readership. A readership is defined by such factors as education, class and age.] 43. Who among the following presented the concept of ‘multi-accentuality’ of the sign, saying that signs possess an ‘inner dialectical quality’ and ‘evaluative accent’? (1) Ronal Barthes (2) Stuart Hall (@)acques Derrida (4)ValentinVoloshinov Answer: (4)[In his famous book Marxism and the Philosophy of Language (1929) ValentinVoloshinov presented his concept of ‘multi-accentuality’ of the signs. In Voloshinov’s view, the meaning of verbal signs is the arena of continuous class struggle: a ruling class will try to narrow the meaning of social signs, making them “uni- accentual”, but the clash of various class-interests in times of social unrest will make clear the “multi-accentuality”” of words. Verbal meanings are subject to historical changes and are a site of struggle between different social groups in particular socio-historical contexts. Language is never neutral. “Whenever a sign is present, ideology is present, too.’ Dominant groups seek to fix meanings: they seek to decide what constitutes ‘extremism’, for instance. ] 44, Arrange the following authors in the chronological order of their birth: A. Oscar Wilde B. William Langland C. Geoffrey Chaucer D. John Dryden E. Alexander Pope ‘Choose the correct answer from the options given below: ()B,C,D,E,A, (QA,B,C,E,D, @)B,C.D,A.E (4)C,B,A,E,D Answer: (1)[William Langland was born in 1330 and died 1400. Geoffrey Chaucer was born in 1342/43 and died on October 25, 1400. John Dryden was born on August 9, 1631 and died on May 1, 1700. Alexander Pope was 15 born on May 21, 1688 and died on May 30, 1744. Oscar Wilde was born on October 16, 1854 and died on November 30, 1900.] 45, Which one of the following Sherlock Holmes stories refers to a significant event in English history? (1) “The Musgrove Rituals” (2) The Speckled Band” (3) “The Solitary Cyclist” (4) “The Red Headed League” Answer: (1) [The Adventure of the Musgrave Ritual” is a short Sherlock Holmes story written by Arthur Canon Doyle, narrated by Holmes himself to Watson. The story is associated with English Civil War and Crowning of Charles II. In an incident in the story Holmes cleans up the metal and stones, and reveals a gold crown, deducing that it had once been a crown of King Charles I, as the ritual dates from that period.It was also possible that a ‘Musgrave had been entrusted with the crown during the English Civil War, but had been murdered before King Charles I had been crowned.} 46. Which book of Paradise Lost incorporates the speech rhythm of Adam and Eve's marital quarrel? (1) Book 4 (2) Book 6 (3) Book 7 (4) Book 9 Answer: (4) [Milton s Paradise Lost is the story of Adam and Eve. It deals with their temptation, fall and repentance. In Book 9, Adam quarrels with Eve and says that if only she had listened to him this morning and if they had done their work together, she would not have been tempted and they would not be in a condition where they are right now. Eve fights back. She says that the serpent was so tricky, cunning, and deceptive that even had it been Adam that he had targeted, he would have been successful in his temptation. Adam ends book 9 by telling Eve that she was warned, told of the dangers that were waiting for them, but she didn’t listen. Adam tries to shift the blame from him to Eve, saying that if Eve had only listened to Adam, she might not have eaten the fruit from the Tree] 47, Arrange the following 18 century magazines in the chronological order of publication: A. The Critical Review B. The Monthly Review C. The Gentleman's Review D. The Rambler Choose the correct answer from the options given below: ()A,D,B,C (2)D,A,B,C (@)B,A,C,D (4)CB,D,A Answer: (4)[The Gentleman's Magazine was a monthly magazine founded by Edward Cave in January 1731. ‘The Monthly Review (1749-1845) was an English periodical founded by Ralph Griffiths. The Rambler was a periodical started by Samuel Johnson. It was published on Tuesdays and Saturdays from 1750 to 1752 and totals 208 issues. The Critical Review was a Magazine appearing from 1756 to 1817. Itwas first edited by Tobias Smollett.] 48, Who among the following feminist theorists posited a separate realm of female experience captured in asstyle of writing different from men’s? A.Elaine Showalter B. lucelrigaray C. Kate Millet D, Simone de Beauvoir E, Helen Cixous Choose the correct answer from the options given below: (A, C, and D only (2)B and D only (8)C, D, and E only (4)B and E only Answer: (4) [Luce Irigaray is a philosopher,psychoanalyst and linguist, whose criticism of patriarchal production of meanings and subjectivity has been among the mostinfluential in French post-war feministwritings. Although her works have at times been dismissed by Anglo-Americantheorists, whether because of what theyperceive as her ‘essentialist’ position onsexual difference or because of her poetic, even cryptic, writing style, the significance of her intervention in patriarchal discoursesacross a wide range of disciplines started to receive ‘more recognition in the 1990s. Irigaray became widely known to Anglo-American critics with Speculum of the Other Woman (1974) and This Sex Which Is Not One (1977; trans. 1985),where she criticizes phallocentrismin Wester philosophical discourse, especiallyin psychoanalysis. HeleneCixusis a feminist poststructuralist, literary critic, poet, novelist and playwright. Cixous is most famous for her essays ‘Sorties’ and “The Laugh of the Medusa’ which explore the links between female sexuality, the psyche and language. Her main theoretical concern is expressed in ‘Sorties’ where she attempts to find ‘ways out’ of a masculine phallocentric binary system that excludes the ‘feminine’ and exiles women from their bodily pleasures.] 49. Which two poems in the following list are examples of dramatic monologue? A. Alfred Tennyson, “Ulysses” B. Philip Larkin, “Church Going” C. Carol Ann Dufty, “Medusa” D. Katherine Philips, “A Married State” Choose the correct answer from the options given below: (A and D only (2)BandC only (3)CandD only (4)A and C only Answer: (4)[Dramatic monologue means self-conversation, speech or talks which includes interlocutor presented dramatically. It means a person, who is speaking to himself or someone else speaks to reveal specific intentions of his actions. Alfred Tennyson, ‘Ulysses ’(1855) and Carol Ann Duffy, ‘Medusa’ (1999) are examples of dramatic monologue. 50. Who makes the following speech in Samuel Beckett’s Waiting for Godot? “Astride of a grave and a difficult birth. Down in the hole, lingeringly, the grave-digger puts on the forceps.” (1) Estragon (2) Lucky GB) Vladimir (4)Pozzo Answer: (3)[The most famous and most critically acclaimed work Samuel Beckett's Waiting for Godot was produced in 1953. The setting is sparse, almost vacant, and the characters are two tramps, Vladimir and Estragon, who do little except wait, on two successive nights, for someone who never appears. While waiting they engage ina series of apparently random discussions, some involving philosophy, and a variety of antics—from taking off their shoes to eating a carrot—that seem vaguely reminiscent of a comedy routine or a vaudeville act. At the end of the play Vladimir offers a grim vision of human life saying: “Astride ofa grave and a difficult birth, Down in the hole, lingeringly, the grave digger puts on the forceps.”] 51. Which of the following are the major themes in William Congreve’s The way of the World? (1) Jealousy and Revenge (2) Love and intrigue (3) Intrigue and death, (4) Love and loyality Answer: (2) [William Congreve’s Way of the World (1700) is the delicate handling of the love game as played by Mirabell and Millamant. They represent the ideal of the Restoration attitude, intense yet balanced, their love based on mutual esteem with no surrender of individuality. Everyone is engaged in intrigue: Mirabell intrigues to gain consent to his marriage from Lady Wishfort, and this involves intrigue within intrigue, for he does not trust NTA UGC NET 2020 (Morning Shift, Paper-II) 17 Waitwell. Fainall intrigues in turn, Everyone is involved in one or the other of these schemes — Mrs. Fainall, Mrs. Marwood, and the servants. Even Lady Wishfort in her willingness to marry Sir Rowland has a devious purpose — revenge on Mirabell. When Mrs. Fainall married her husband that was part of an intrigue, as was his marriage to her. And as we see in the play, victory goes to Mirabell, not because of his virtue, but simply because he is the ‘most successful intriguer.] 52. Arrange the following plays in the chronological order of publication: A. All for Love B. Venice Preserved C. The School for Scandal D. The Country Wife ‘Choose the correct answer from the options given below: ()B,C,A,D (2)D, A,B,C @)CB,D,A (4)A,D,C,B Answer: (2)[The Country Wife is a Restoration comedy written in 1675 by William Wycherley. All for Love isa 1677 heroic drama by John Dryden. Venice Preserv'd is an English Restoration play written by Thomas Otway. It was first staged in 1682. The School for Scandal is a comedy of manners written by Richard Brinsley Sheridan. twas first performed in London on 8 May 1777,] 53. A research hypothesis is: A. Aproposition which is always true B.A provisional explanation of anything C. a theory which will be disapproved by evidence D_A statement which is assumed (o be true for the sake of argument. Choose the most appropriate answer from the options given below: (1)Aand B only (2) Band Conly (@)BandD only (4)Aand C only Answer: (3) [Hypothesis is usually considered as the principal instrument in research. Its main function is tosuggest new experiments and observations. A hypothesis may be defined as a proposition ora set of proposition. According to the Chambers Twentieth Century Dictionary, “Hypothesis is a supposition, a proposition assumed for the sake of argument, a theory to be proved or disproved by reference to facts, a provisional explanation of anything”. The Oxford Reference Dictionary defines a hypothesis as “a proposition or supposition made from known facts as the basis for reasoning or investigation”. According to the Merriam-Webster Dictionary, “A. hypothesis is an assumption, an idea that is proposed for the sake of argument so that it can be tested to see if it might be true”.] 54. On December 11, 1823, Rammohan Roy addressed a letter to the British authority which pleaded for modern western education and is considered historically important for the introduction of English education in India. Who was the letter addressed to? (1) Lord Amherst (2) Lord Minto (3) Lord Macaulay (4) Lord Bentick Answer: (1) [On11* December 1823 Rammohan Roy wrote to Lord Amherst that education of Sanskrit language and Sanskrit literature would do nothing and had no practical use. Rather he requested him to promote the western education in India, He felt that the youths could not adapt themselves according to the changing societies if they cling to the age-old Vedantic philosophy or doctrines. Ram Mohan was the chief advocate of the modern process of education and the scientific learning.] 18 BRUSHUPYOUR ENGLISHLITERATURE 55, Match List-I with List-I1 List 1 List-IT Word Borrowed Source Indian Language ‘A. Mangoose 1Tamil B.Loot TI. Malayalam C.Cunry TI. Hindi/ Urdu D.Betel TV. Marathi Choose the correct answer from the options given below: ()ACIV,B- Il, C-I, D-IL (Q)A-IVB-I, C-I, D-II (3)A-Il,B-IIL, C-IV,D-1 (4)A-I1,B-1, C-1V, D-II Answer: (1) [Mongoose is taken from Marathi word Mangus. The term is used to identify the carnivorous animal known for its ability to kill poisonous snakes. Loot is taken from Hindi word Lut which again, has an origin from Sanskrit word Lotra. CurryistakenfromTamilwordKarri, Betel is taken from Malayalam language.] ‘56. Who is the author of “A Fragment” (1819), one of the earliest vampire stories in English? (1)P.B. Shelley (2) Lord Byron (3) Bram Stoker (4) Mary Shelley Answer: (2)[*Fragment ofa Novel” is an unfinished 1819 vampire horror story written by Lord Byron. The story, also known as “A Fragment” and “The Burial: A Fragment”, was one of the first in English to feature a ‘vampire theme] 57. Which one of the following best explains the term ‘paralanguage’? (1) The way in which people mask what they mean by words they use (2) The way in which people show what they mean other than by the word they use (3) The way in which people carry meanings unintended by speaker (4) The way in which the silence underlying speech communicates wrong meaning Answer: (2) [Paralanguage refers to the nonverbal elements of speech ~ such as vocal pitch, intonation, and speaking tempo ~ that can be used to communicate attitudes, convey emotion, or modify meaning. In simple terms, paralanguage can be thought of as how something is said rather than what is said.] 58. Who among the following was the first Director of Central Institute of English and Foreign Languages, Hyderabad (nowEFL University)? (1)Prof. V.K. Gokak (2)Prof. C.D. Narsimhaiah (3) Prof.C.J. Daswani (4) Prof. K.R.S. Ivengar Answer: (3) [The English and Foreign Languages University, Hyderabad, was founded in 1958 as the Central Institute of English (CIE). The mandate of the University is to advance and disseminate “instructional, research and extension facilities in the teaching of English and foreign languages and literatures in India” as well as “to take appropriate measures for interdisciplinary studies and research in literary and cultural studies, and to develop critical intercultural understanding of civilizations.” Prof. V. K. Gokak was the first director of CIEFL.} 59. Arrange the following critical works in their chronological order of publication: A. “Preface to Lyrical Ballads” B.A Defence of Rhyme C. “Life of Cowley” NTA UGC NET 2020 (Morning Shit Paper-I1) 19 D. “The Frontier of Criticism” Choose the correct answer from the options given below: ()A,C,B,D Q)B,A,C,D @)B,C,A,D (@)CA,B,D Answer: (3) []Samuel Daniel’s The Defense of Rhyme was published in1503.Samuel Johnson’s Life of Cowleyfrom Samuel Johnson’s Lives of the Poets series, published between 1779 and 1781. The Preface to the Lyrical Ballads (1800) is an essay by William Wordsworth, for the second edition of the poetry collection Lyrical Ballads, and then greatly expanded in the third edition of 1802.T.S. Bliot’s “The Frontier of Criticism” was published in 1956,] 60. Given below are two statements: one is labeled as Assertion A and the other is labeled as Reason R Assertion A: The introduction of English in India was primarily for the benefit and consolidation of British power. Reason R: English created to the social and economic aspirations of the emerging middle class and urban elites in India. In the light of the above statements, choose the correct answer from the options given below: (1) Both A and R true, and R is correct explanation of A (2) Both A and R true, and R is not correct explanation of A (3) A is true Ris false (4)A is false but R is true Answer: (2) [Both statements are correct but not logically connected.} 61. Arrange the following women novelists in chronological order (by date of birth): A. Anne Bronte B, Jane Austen C.Ann Redeliff D. Fanny Burney E. Maria Edgeworth Choose the correct answer from the options given below: ()B,A,D,C,E QCD,BEA @)D.CEBA @)A,B,C,E,D Answer: (2)[The most representative of English Gothic novelists Ann Radeliffe was born on July 9, 1764 and died on February 7, 1823. Fanny Burney was born on June 13, 1752 and died on January 6, 1840, London.Jane Austen was bom on December 16, 1775 and died on July 18, 1817.Maria Edgeworth was bom on January 1, 1767 and died on May 22, 1849. Anne Bronte was born on January 17, 1820 and died on May 28, 1849.] 62. Given below are two statements: Satement I: Consumption is an outcome of self-interest and a maximization of personal pleasure, Statement II: There are strong correlations between social status and such things as housing styles, musical tastes and food preferences. In the light of the above statements, choose the correct answer from the options given below: (1) Both statement I and IT are true. (2) Both statement I and Il are false (3)Statement Lis correct bur statement IT is false (4) Statement [is incorrect bur statement IT is true Answer: (1) [Consumption is about maximization of personal pleasure. Pleasure might be a componentof aesthetic appreciation or non-aesthetic appreciation. Compare reading a well-loved book while drinking wine. The first case has a psychological component and will result in aestheticappreciation; the second case may bring the 20 BRUSH UPYOUR ENGLISH LITERATURE joy as well, but it has no psychological component —itjust outcomes from the effect of the opiate on the cerebrum, One could likely build up this feeling byfast driving a car; it is enjoyable but without a sense of beauty. Pleasure may be associated withthings as musical tastes or food preferences. ] 63. Which one of the following is correct about Saussure’s analysis of language? (1) La langue is a system of language. (2) Parole focuses on language as a system at a particular time, (3) La langue is the particular instance of speech and writing. (4) Parole is study of language over a period of time Answer: (1) (Deriving from the French la langue meaning tongue, langue refers to a language in its entirety, at any one point in time, and includes the rules and conventions of its use ~ rules which pre-exist individual users.It is this determining element of langue’s characteristic that marks its active nature. In contrast, parole, translated from the French la parole, meaning speech or word, refers to individual utterances of written or spoken language that passively adhere to the rules of the langue. Ultimately, the distinction between langue and parole is a distinction between code and message, structure and performance.] 64, What game do the characters play in Act II of Harold Pinter’s The Birthday Party? (1) A game of Chess (2)Agame of cards (3) Blind man’s buff (4) Musical chair Answer: (3) [In the second Act of Harold Pinter’s The Birthday Partythe characters play blind man’s bluff specifically because it makes them nostalgic, but the sinister side of such nostalgia is inescapable in the stage image of Stanley preparing to rape Lulu, Nostalgia is lovely to feel, the play seems to suggest, but more insidious in its complexities.) 65. The deductive method differs from inductive method in drawing its conclusion from: (1) Verification (2) Particular instances (3) Applications (4) General truths Answer: (4)[The main difference between inductive and deductive reasoning is that inductive reasoning aims at developing a theory while deductive reasoning aims at testing an existing theory.Inductive reasoning moves from specific observations to broad generalizations, and deductive reasoning the other way around.] 66. Which two of the following are best evoked by Hamlet's utterance- “To be or not to be”? A. between life and death B. Between action and emotion C. Between affirmation and confirmation D. Between doing and abstaining from doing Choose the correct answer from the options given below: (A and D only (2)B and D only (3)Cand A only (4)D and C only Answer: (1) [One of the best soliloquies in English literature “To be or not to be” is Hamlet’s contemplation about life and death. It signifies “To live omot to live” (or “To live or to die”). Hamlet examines how excruciatingly painful and hopeless humanlife is, and how death (specifically suicide) would be preferable, would it not be for the frightfuluncertainty of what comes after suicide. The quote “To be or not to be” also signifies “Action vs. Inaction”. Hamlet wrestled with whether to actor not. It’s perhaps one of the greatest challenges we all face every single day: what to do; when to doit; never sufficient opportunity or time. And the tormenting questions about how will itbe received.regardless of whether what we do is good enough or not.] NTA UGC NET 2020 (Morning Shift, Paper-Il) 21 67. Match List- I with List- I List List-I Essayist Essay ‘A. George Orwell T.“On the Artificial Comedy of the Last Century” B. Michel de Montaigne TL. “Why I Write” C. Charles Lamb TIL. “A Modest Proposal” D. Jonathan Swift TV. “On the Cannibals” Choose the correct answer from the options given below: ()A-IIL,B-IV,C-1, D-II (Q)A-IIB-IV,C-1,D-II @)A-IV, B-IIl, C-Il, D-I (4) A-I,B-IIl,C-1,D-IV Answer: (2)[Charles Lamb’s essay "On the Artificial Comedy of the Last Century” (1822) both helped to revive interest in Restoration comedy and anticipated the assumptions of the Aesthetic movement of the late 19th century. “Why I Write” (1946) is an essay by George Orwell detailing his personal journey to be a writer. A Modest Proposal is a satirical essay written and published anonymously by Jonathan Swift in 1729. “Of Cannibals” is an essay by Michel de Montaigne, describing the ceremonies of the Tupinamba people in Brazil] 68. Given below are two statements: one is labeled as Assertion A and the other is labeled as Reason R Assertion A: Signs are neutral or innocent. Reason R: In all cases signs are organized into system that convey some meaning In the light of the above statements, choose the correct answer from the options given below: (1) Both Aand R true, and R is correct explanation of A (2) Both A and R true, and Ris not correct explanation of A (3) Ais true R is false (4)Ais false but R is true Answer: (1)[Signs are never neutral or monoglossic. The potential power of signs is directly proportional to the level of intercultural ignorance. In some cases, that power of signs is asymmetrical and therefore intolerable. Signs are never neutral but infused with the sign-creator’s inclinations and interests. Signs are never neutral, but always socially and culturally produced and arranged, and motivated. Signs are never neutral, but rather directed to a particular mentality, or approach] 69. Harold Skimpoleis a character in (1) Bleak House (2)Dombey and Son (3) Great Expectations (4)Oliver Twist Answer: (1) [In Charles Dickens's Bleak House (1853) Harold Skimpole isa friend of Mr. Jamdyce, who calls, himself a “child” and claims to have no idea about time or money. Mr. Skimpole borrows money liberally with no thought of repaying it. He eventually betrays Mr. Jarndyce by telling Inspector Bucket that Jo is in the stable at Bleak House.] 70. According to his essay “Civil Disobedience”, what two things did Thoreau learn from the night he spend in jail? ‘A. He conchuded that the state is ultimately weak B. He realized that captivity inspires courage C. He realized that neighbours are only friends during good times. 22 BRUSHUPYOURENGLISH LITERATURE D. He concluded that captivity brings wisdom about human affairs ‘Choose the correct answer from the options given below: (1) Aand B only (2)AandC only (3) Aand D only (4) CandD only Answer: (2)[Civil Disobedience is an essay by American transcendentalist Henry David Thoreau that was first published in 1849. In his essay, Thoreau observes that only a very few people — heroes, martyrs, patriots, reformers in the best sense ~ serve their society with their consciences, and so necessarily resist society for the ‘most part, and are commonly treated by itas enemies. Thoreau, for his part, spent time in jail for his protest. By his experience he concluded that the state is ultimately weak and realized that neighbours are only friends during good times. Thoreau’s technique of passive resistance, as described in ‘Civil Disobedience’, was adopted by Gandhi] 71. Which two of the following inspired the rise of periodical essays? ‘A. Robert Burton B. Francois Rabelais C. Francis Bacon D. Michel de Montaigne Choose the most appropriate answer from the options given below: (1)Cand A only (2)A andB only (3)CandD only (4)B and D only Answer: (3) [Francis Bacon is generally credited with introducing and popularizing ‘Formal’ essays in theEnglish-speaking world. Bacon was influenced by the French essays of Michel de Montaigne. Montaigne first used the term “essais” (or “attempts”) to describe his prose reflections on ordinarytopics.] 72. Arrange the following in the chronological order of publication A. Aspect of the theory of Syntax B. Course in General Linguistics C. Semiotics and philosophy of Language D. How to do things without Words. Choose the correct answer from the options given below: ()D,B,A,C (2)C,B,A,D @)B,D,A,C (4)B,A,D,C Answer: (3) [Course in General Linguistics (1916) isa book compiled by Charles Bally and Albert Sechehaye. How to Do Things with Words (1955/1962) is perhaps J. L. Austin’s most influential work. Aspects of the Theory of Syntax is a book on linguistics written by Noam Chomsky, first published in 1965. Umberto Eco’s Semiotics and the Philosophy of Language (1984) comprehends the entire tradition of the doctrine of signs, threading its way through the symbolic and allegorical readings. ] 73.As mentioned in “My First Acquaintance with Poets” which poet does William Ha: “only person I ever knew who answered the idea of a man of genius”? (1) Coleridge (2) Wordsworth (3) Byron (4) Shelley Answer: (1)[In My First Acquaintance With PoetsHazlitt describes his factual meeting with poets, “William Wordsworth and Samuel Coleridge”. Set in 1798, he describes how he traveled under harsh weather conditions on foot just so he could hear the poet, Coleridge. In his depiction Coleridge was a man who had the power to paint imagery with words. Hazlitt was so impressed by his way of preaching that he ended up fixing a meeting with the author. At this meeting her encountered William Wordsworth as well. Hazlitt wrote, “But I may say of him here, that he is the only person I ever knewwho answered to the idea of a man of genius. He is the only person from whom. Jever learn anything.”} 74, Who among the following is known to have popularized the term ‘glocalization”? (1) Ronald Robertson (2) Francis Fukuyama (3) John Urry (4) John Tomlison describe as the NTA UGC NET 2020 (Morning Shift, Paper-II) 23 Answer: (1) [Glocalization is a combination of the words “globalization” and “localization.” Itis the adaptation of global and intemational products, into the local contexts they're used and sold in. The term was coined in the Harvard Business Review, in 1980, by sociologist Roland Robertson, who wrote that glocalization meant “the simultaneity—the co-presence—of both universalizing and particularizing tendencies.”] 75. Which of the following assumptions best express the position of Post- Structuralist criticism? (1) Definite structures underlie empirical events. (2) Language is representational (3) Apprehension of reality is construct. (4) Knowledge operates according to procedures that are axiomatic Answer: (3)[The term Poststructuralisin is used to describe those kinds of thinking and writing that disturb orexceed the ‘merely’ rational or scientific, self-assuredly ‘systematic’ work of structuralists.Itis primarily associated with the work of Derrida, Lacan, Foucault, Deleuze and Guattari, Cixous and (post-1967) Barthes. Poststructuralism entails a rigorous and, in principle, interminable questioning of every centrism (logocentrism, ethnocentrism, anthropocentrism, etc.), of all origins and ends, meaning and intention, paradigm or system.According to Post- structuralist criticism: Apprehension of reality is a construct. There are three basic assumptions of Structuralism: 1. knowable structures underlie empirical events, 2. knowledge operates according to procedures that are axiomatic and not open to question, and 3. reality is not radically contingent, not a play of forces without order or a series of accidents or events without meaning ot logical sequence.] 76, Which of the following novels is structured into a poem of 999 lines, preceded by a Foreword followed bya Commentary and an Index? (1) Ragtime (2) Pale Fire (3) The Inner Side of the Wind (4) Hourglass Answer: (2) (Pale Fire (1962) is a novel by Vladimir Nabokov. It consists of a long poem of 999 lines in heroic and a commentary on it by an insane pedant. This brilliant parody of literary scholarship is also an experimental synthesis of Nabokov’s talents for both poetry and prose. It extends and completes his mastery of unorthodox structure. 1. Which two of the following strictly follow the parameters of documentation prescribed by the eighth edition of the MLA Handbook? A. Nunberg, Geoffery, editor. The Future of the Book, U of California P, 1996 B, Puig, Manuel, Kiss of the Spider Woman. Trans. Thomas Colchie, London; Vintage, 199(11) C. Nunberg, Geoffery, ed. The Future of the Book Berkeley: U of California P, 1996. D. Puig, Manuel. Kiss of the Spider Woman. Translated by Thomas Colchie, Vintage Books, 199(1) Choose the correct answer from the options given below: (1) AandB only (2)AandC only (3)Aand D only (4) Band C only Answer: (3) [MLA stands for “Modern Language Association”, which is based in the United States. MLA Handbookprovides guidelines for documenting sources in scholarly writing.] 78. To which mythological character is Faustus compared in the Prologue of Dr. Faustus? (1) Perseus (2) Theseus (3) Icarus (4) Achilles Answer: (3) [The prologue connects Doctor Faustus to the Greek legend of Iearus. Theprologue compares the temperament and experience of the protagonist Faustus’ to Icarus” ruinedeffort to fly. Icarus was a boy whose father, Daedalus, gave him wings made out of feathers and beeswax andalso warned him not to fly high. Icarus didn’t pay any attention to his father’s warning and flewextremely close to the sun, causing his wings to meltdown 24 BRUSHUPYOURENGLISHLITERATURE, and sending him plunging to his death. Similarly, the Chorus lets us know, Faustus will “mount above his reach” and experience the samedestiny.] 79. Who said of the blank verse quoting an unnamed critic that it is “...verse only to the eye”, adding further that it “has neither the easiness of prose, nor melody of numbers”? (1) John Dryden (2) Alexander Pope 3) S.T. Coleridge (4) Samuel Johnson Answer: (4) [In his Life of MiltonSamuel Johnsonwrotes, “The variety of pauses, so much boasted by the lovers of blank verse, changes the measures of an English poet to the periods of a declaimer; and there are only a few skilful and happy readers of Milton who enable their audience to perceive where the lines end or begin. “Blank verse,” said an ingenious critic, “seems to be verse only to the eye.”Poetry may subsist without rhyme, but English poetry will not often please, nor can rhyme ever be safely spared but where the subject is able to support itself. Blank verse makes some approach to that which is called the “lapidary style”; has neither the easiness of prose nor the melody of numbers, and therefore tires by long continuance] 80. Which one of the following statement is true about Aristotle’s poetics? (1) He asserted the value of poetry by integrating rhetoric and imitation (mimesis) (2) He asserted the value of poetry by focusing on both rhetoric and imitation (mimesis) (3) He asserted the value of poetry by giving preference to rhetoric and imitation (mimesis) (4) He asserted the value of poetry by focusing on imitation (mimesis) rather than rhetoric. Answer: (4) [Mimesis is a concept originally developed by Aristotle within the context of the atrical tragedy, mimesis is essentially concerned with how art imitates reality. Such imitation involves the display or presentation. of action rather than the imaginative concept of action, which is termed diegesis. In other words, to imitate an action and present it asreal is mimetic, whereas the imagination of an action is diegetic. According to Aristotle, mimesis involves the representation of reality, in particularwith regards to human emotions rather than human intellect. Aristotle was particularly concerned with how the concept of mimesis functions in tragedy and sought to show how drama was an imitation of reality.] 81. Which British Administrator sought “to make everything as English possible in a country which resembles England in nothing,” as recorded by Sir Thomas Munro? (1) Lord Bentick (2) Lord Hastings (3) Lord Camwallis (4) Lord Welesley Answer: (3)[Comwallis sought to implant English institutions in India. Thomas Munro recorded that ‘Comwallis sought “to make everything as English as possible in a country which resembles England in nothing”,] 82. Which two of the following events are described in Samuel Pepys’s Diary? ‘A. The Plague in London B. The Great Fire of London C. The War of Spanish Succession D. Essex Rebellion ‘Choose the correct answer from the options given below: (1)Aand B only (2)Aand Conly (3) Band Conly (4) Band D only Answer: (1) [Samuel is famous for the diary he kept for a decade. Pepys began to keep a diary from | January 1660. The last entry was made on 31 May 1669. His diary reveals his jealousies, insecurities, trivial concerns, and his fractious relationship with his wife. Itis an important account of London in the 16608, The juxtaposition of his commentary on politics and national events, alongside the very personal, can be seen from the beginning. It provides a combination of personal revelation and eyewitness accounts of great events, such as the Great Plague NTA UGC NET 2020 (Morning Shift, Paper-II)_ 25 of London, the Second Dutch War and the Great Fire of London.] 83.The Duchess of Malfiis based on: (1) AFrench romance (2)An Italian novella (3)AGerman fable (4) A Scottish chronicle Answer: (2) [Webster wrote The Duchess of Malft in 1612/13. The play is based on an Italian novella, which in tur is based on true historical events. The real Duchess, Giovanna d’Aragona, married Antonio Beccadelli in secret and bore him three children. She was murdered by her brothers, one of whom was a Cardinal, in 1510. ‘Webster’s main changes to the true story are that Antonio didn’t die until a few years after the Duchess’ death, and Bosola’s repentance and ultimate betrayal and murder of the brothers is fictionalized, as the two were never accused or killed.] 84. Which one of the following essays holds that “As a method, realism is a complete failure”? (1) Virginia Woolf, “The Mark on the Wall” (2) Oscar Wilde, “The Decay of Lying” (3) D. H. Lawrence, “Why Novel Matters” (4) Mary McCarthy, “My Confession” Answer: (2)[Oscar Wilde “The Decay of Lying” proves his promotion of Romanticism over Realism. Wilde writes, “The moment art surrenders its imaginative medium it surrenders everything. As a method, realism is a complete failure, and the two things that every artist should avoid are modernity of form and modemity of subject matter.”] 85, Given below are two statements: Statement I: The Orientalists in British India were not sympathetic towards India’s ancient learning. Statement IT: William Jones thought that an “imagination”, “ratiocination” and philosophy, Indians are no means inferior to Europeans. In the light of the above statements, choose the correct answer from the options given below: (1) Both statement I and Il are true. (2) Both statement I and Il are false. (3) Statement I is correct bur statement IT is false (4) Statement I is incorrect bur statement II is true Answer: (4) [The Orientalists in British India were very eager to know India’s ancient learning. William Jones ‘was one of them and he had the opinion that Indians are no means inferior to Europeans.] 86. Which two terms among the following are associated with the formalist criticism? A. Aura B. Actant C.Narratee D, Defamiliarization E. Foregrounding ‘Choose the correct answer from the options given below: (1) Aand C only (2)BandD only (3)BandC only (4) DandE only Answer: (4)[Formalist criticism refers generally to kinds of criticism that emphasize the importance of the formal dimensions of literary texts, such as prose style, rhyme, narrative structure, verse-form and so on. In this, respect formalism is seen to stress the importance of form as (supposedly) distinct from content, meaning, social and historical context, ete. ‘The Russian formalist critic Viktor Shklovsky uses the term ostranenie, usually translated as ‘making strange’ 26 BRUSHUPYOUR ENGLISH LITERATURE, or ‘defamiliarization’, to denote what he sees as the primary function of literary texts ~ to make the familiar unfamiliar, to renew the old, or make the habitual appear fresh or strange. According to formalist critics, foregrounding, isa stylistic device that hypothetically promptdefamiliarization, evoke feelings, and prolong reading time.] ‘87. Who among the following are two great masters of the French language that T. S. Eliot contrasts with Dryden and Milton in ‘The Metaphysical Poets’? A. Francois Villon B. Jean Racine C. Charles Baudelaire D. Arthur Rimbaud Choose the correct answer from the options given below: (1)AandC only (2)Aand D only 3) BandConly (@) Band D only Answer: (3) [Eliot’s influential essay “The Metaphysical Poets” (1921) appeared as a review of Herbert J.C. Grierson’s anthology Metaphysical Lyrics & Poems of the Seventeenth Century. In this essay Eliot argued that the works of these men embody a fusion of thought and feeling that later poets were unable to achieve because of a “dissociation of sensibility,” which resulted in works that were either intellectual or emotional but not both at once. Eliot discusses Milton and Dryden, and their influence was most unhealthy, because as a result of their influence there set in a ‘dissociation of sensibility’ from which English poetry has recovered only in one modem age. Both Milton and Dryden were in contrast to two great masters of French language—Jean Racine and Charles Baudelaire.] 88. Match List-I with List- 1 List List-I Crities text ‘A. Horace 1.A Defence of Rhyme B. John Dryden I. Timber, or, Discoveries C. Samuel Daniel Ill. ArsPoetica ‘D. Ben Jonson TV. Of Dramatic Poesy ‘Choose the correct answer from the options given below: ()A-H,B-1,C-1V,D-IIT Q)A-TIB-IV,C-H,D-1 (G)A-II,B-IV,C-I, D-II (@)A-HLB-IV,C-1,D- IIL Answer: (3) [4rs Poetica is a poem written by Horace in 19 BC in which he advises poets on the art of writing poetry and drama, Ben Jonson’s Timber is a collection of meditations and commentaries upon a range of issues, from the nature of fortune, fame, opinion and wisdom to observations on dramatic theory and poetics. The Defense of Rhyme (1503) is written by Samuel Daniel. John Dryden’s An Essay on Dramatic Poesy presents a brief discussion on Neo-classical theory of Literature. ] 89. Which two of the following statements are applicable to ‘metalanguage”? Itis: A. a technical language which describes the properties of language. B. Known as the ‘first order’ language. C. a ‘second order” language that replaces a ‘first order’ language with metaphors. NTA UGC NET 2020 (Moming Shift, Paper-II) 27 D.a ‘second order’ language. ‘Choose the correct answer from the options given below: (1) Aand B only (2)Cand D only (3) Aand D only (4) Band C only Answer: (3)[Metalanguage is a concept that has beenpresent in Western thought since the pre- Socraties, but was of particular concern to structuralism and the work of, amongstothers, Roman Jakobson. In linguistics it is used to describe the way in which one language talks about another language (the object-language). For example, the object language may be a literary text and the metalanguage a commentary on it. Within the domain of critical theory the term was popularized in 1974 in the film journal Screen by Colin MacCabe. His Reatism and the Cinema begins by describing the way in which the nineteenth-century classical realist novel organizes its discourse with regards to the metalanguage and ‘truth’.] “Gr novel. 90. Match List-I with List- IT List List-I Author ‘Text ‘A. Thomas Pynchon LG B. Howard Jacobson ILV C. Anthony Burgess J D. John Berger IV.MIE Choose the correct answer from the options given below: ()A-IL,B-IV,C-1, D-IL (Q)A-IIB-IM, C-1V,D-I (@)A-II,B-III, C-1,D-IV (@)A-IV,B-IIL, C-1, D-IL Answer: (3) [G isa 1972 novel by John Berger, set in pre-First World War Europe, and its protagonist, named Vis a novel by Thomas Pynchon, published in 1963 and granted the Faulkner Foundation award for a first Jisa2014 novel by Howard Jacobson, It was shortlisted for the 2014 Man Booker Prize.MIF is a 1971 novel by Anthony Burgess. ‘Comprehension: ‘Read the given passage and answer the questions that follow Daybreak At dawn she lay with her profile at that angle Which, sleeping, seems the stone face of an angel; Her hair a harp the hand of a breeze follows To play, against the white cloud of the pillows. Then in a flush of rose she woke, and her eyes were open, Swimming with blue through the rose fresh of dawn. From her dew of lips, the drop of one word Fell, from a dawn of fountains, when she murmured ‘Darling.’ — upon my heart the song of the first bird. “My dream glides in my dream,’ she said, ‘come true. 28 BRUSHUPYOURENGLISHLITERATURE Iwaken from you to my dream of yo , then my waking dream dared to assume The audacity of her sleep. Our dreams Flowed into each other’s arms. like streams. ~ Stephen Spender 91. Match List- I with List- 1 List List Item ‘What is an example of A.‘Herhaira harp” LSimile B. ‘the hand of a breeze Il. Metaphor C. ‘seems the stone face” TH. Oxymoron, D. My walking Dream IV. Synecdoche Choose the correct answer from the options given below: ()A-I,B-IV,C-1, D-IL (Q)A-IVB-II,C- IIL, D-I ()A-IV,B-IML, C-II, D-T (4)A-1,B-IV,C-11, D-II Answer: (1) 92. Which among the following best describes the lady’s face as “at dawn she lay. (1) Her face appears to be that of a stone sculptures. (2) The side- view of her face appears to be that of a sculpted angel’s (3) Her face appears to be that of a stone- angel. (4) The side- view of her face appears to be that of an angel’s Answer: (2) 93.Match List- I with List- IL List-1 List Item What is an example of A.‘Herhair’ 1. prayer B. ‘pillows’ IL ‘a harp” C. Breeze” TI. “tose” D. “Cheeks” TV. “cloud Choose the correct answer from the options given below: (1)A-1, BIL, C-IV, D-IL (2)A-IIB-1, C-IL,D-IV (3) A-IL,B-IV,C-1, D-IL (@)A-1V,B-IM, C-1,D-IT Answer: (3) Read the given passage and answer the questions that follow: Logic cannot have any empirical part; that is, a part in which the universal and necessary laws of thought rest on grounds taken from experience; otherwise it would not be logic, i.e., a canon for the understanding or the reason, valid for all thought, and capable of demonstration. Natural and moral philosophy, on the contrary, can NTA UGC NET 2020 (Morning Shift, Paper-II) 29 each have their empirical part, since the former has to determine the laws of nature as an object of experience; the letter, the laws of human will, so far as itis affected by nature: the former, however being laws according to which everything does happen; the letter, laws according to which everything ought to happen. Bthics, however, must also consider the conditions under which what ought to happen frequently does not. — Immanuel Kant. 94. “Logic cannot have empirical part” because: A. laws of thought are subjective B. It propounds laws whose applicability can be shown. C. Its laws are valid for all thought. D. Its laws are valid for everyone’s experience. Choose the correct answer from the options given below: (1) Aand D only (2)BandC only (3) Aand C only (4) BandD only Answer: (2) 95. Based on the passage which two of the following statements are correct? A. For natural philosophy, nature influences the laws. B. For moral philosophy, nature is to be experienced. C. Natural philosophy does not describe how things actually do happen. D. Moral philosophy accounts for what should be. Choose the correct answer from the options given below: (1) AandConly (2)BandD only (G)CandD only (4)AandD only Answer: (4) Read the given passage and answer the questions that follow: And the creature run from the cur? There thou mightst behold the great image authority; a dog’s obeyed in the of Thou rascal beadle, behold, hold thy bloody hand! Why dost thou lash that whore? Strip thine own back; Thou hotly lust’st to use her in that kind For which thou whipp’st her. The usurer hangs the cozener. Through tatter'd clothes small vice do appear; Robed and furr’d gown hide all, Plate sin with gold, And the strong lance of justice heartless breaks; Armas in its rags, a pigmy’s straw doth pierce it. King Lear 96. Who speaks these lines and to whom? (1) Edgar to Lear (2)Goneril to Edgar (3) Lear to Gloucester (4) Gloucester to Lear Answer: (3) 97. In the passage, the church officer is asked to whip his own back rather than the prostitute’s because: (1) Asa religious man he should punish himself for other’s sins. (2) He at one time had lusted after her. (3) Men like him make them prostitutes. 30 BRUSHUPYOURENGLISH LITERATURE (4) He does not have any authority to whip a woman, Answer: (3) 98. The two sentences from the lines from “through tatter’d clothes...” to“... straw doth pierce it” deal with two foibles, (i) vice and (ii) sin. About these two, the speakers says that (1) Vice afflicts all sin afflicts only the weak (2) Sin afflicts all but vice afflicts only the strong. (3) Sin and vice are seen in both the weak and the strong (4) Sin and vice are palpable in the weak and impalpable in the strong Answer: (4) ‘Comprehension: The surgeon deposited in her arms, she imprinted her cold, white lips passionately on its forehead; passed her hands over her face; gazed wildly around; shuddered; fell back- and died. They chafed her breast, hands, temples; but the blood had stopped forever. They talked of hope and comfort. They had been strangers too long. ‘I's all over, Mrs. Thingummy!”, said the surgeon at last. —Dickens, Oliver Twist 99, The implication of “they had been strangers too long” is: (1) Those who spoke of *hope and comfort had been strangers too long. (2) ‘Hope’ had been stranger to ‘comfort’ for too long. (3) ‘Hope and comfort’ had been stranger to the patient too long, (4) “Hope and comfort’ had been stranger to the surgeon, nurse and the patient too long. Answer: (3) 100. In the expression, “passed her hands over her face”, the ‘face’ is of: (1) the lady surgeon (2) the child (3) the nurse (4) the patient Answer: (4) 2. NTA UGC NET 2020 (Evening Shift, Paper-II) 1. Which of these following statements are true about Pidgin and Creole? (A) Pidgin begins as Creole and eventually becomes first language of a speech community. (B) Creole begins as Pidgin and eventually becomes first language of a speech community. (©) Pidgin is simple buta rule governed language developed for communication whereas Creole in free from grammatical rules. (D) Pidgin and Creole evolve successively out of a situation where speakers of mutually unintelligible languages develop a shared language for communication (often based on one of those languages). Choose the correct answer from the options given below: (1) (A) and (B) only (2)(B) and (D) only (3) © and (D) only (4) (A)and (D) only Answer: (2) [Pidgins are languages serving as lingua franca, that is, they are used as a medium of communication betweengroups who have no other language in common. When ‘two or more people use a language ina variety whose grammar and vocabulary are very much reduced in extent and which is native to neither side” they are using a pidgin. A creole “arises when a pidgin becomes the native language of a speech community, as in the Caribbean’. Pidgins typically develop out of trade languages and may evolve into creoles.] 2.Which of the following short stories by Edgar Allan Poe has a narrator who has a rival with the same name and uncanny physical resemblance? (1) “Hop- Frog” (2) *William Wilson” (3) “The System of Doctor Tarr and Professor Fether (4) “The Imp of Perverse” Answer: (2) [“William Wilson” is a short story by Edgar Allan Poe, first published in 1839, with a setting inspired by Poe’s formative years outside of London. The tale follows the theme of the doppelginger.The story centresaround a man named William Wilson, who is met with another man named William Wilson, with the same age, height, looks and aspects of their life.] 3. To which of these boarding schools is Jane Eyre sent by her aunt Mrs. Reed? (2) Lowood School (2) Hailsham Schoo! (3) Abbey Mount (4) Greyfriar’s School Answer: (1) [Charlotte Bronte’s novel Jane Eyre was published in 1847. Widely considered a classic, it gave new truthfulness to the Victorian novel with its realistic portrayal of the inner life of a woman, noting her struggles with her natural desires and social condition. When the novel begins, the title character is a 10-year-old orphan who lives with her uncle’s family; her parents had died of typhus. Other than the nursemaid, the family ostracizes Jane. She is later sent to the austere Lowood Institution, a charity school, where she and the other girls are mistreated; “Lowood,” as the name suggests, is the “low” point in Jane’s young life. In the face of such adversity, however, she gathers strength and confidence.] 4, Match List- I with List- Il List-1 List Crities Essays (@)L.C. Knights (i) “The Study of Poetry” (b) Lione! Trilling (ii) “Restoration Comedy: The Reality and the Myth” (©) Matthew Arnold Gi) “Poetry for Poetry’s Sake” (@A.C. Bradley (ii) “The Sense of the Past™ Choose the correct answer from the options given below: (1) @Lii), (0-6), (©)-0), (@)-Gi) 2) (apiv), (6), (i), (@)-Gi) GB) @-Gi), ©)-Gv), ()-(i), @-Gi (4) @Hiv), (b)-Gii), (){i), (@)-Gi) Answer: (3) [Matthew Amold’s “The Study of Poetry,” was originally published as the general introduction to TH. Ward’s anthology, The English Poets (1880). L. C. Knight’s Restoration Comedy: The Reality and The Myth was published in 1937. "Poetry for Poetry s Sake” is a lecture delivered by A. C. Bradley on June 5, 1901. “The “Sense of the Past” is an essay by Lionel Trilling, It was published in 1942,] 5. Which one of these following essays by Ezra Pound defines an image as “that which presents an intellectual and emotional complex in an instant of time”? (1) “A Retrospect” (2) “The Tradition” (3) “The Renaissance” (4) “How to Read” Answer: (1) [Ezra Pound’s “A Retrospect” (1918) is a collection of his essays on poetry. In“A Retrospect” Pound presents his beliefs about what makes good poetry. He defines “image” as “an intellectual and emotional complex in an instant of time.” He elaborates on the “rules” of imagism, advising precision, and proclaiming, among other things, “Use either no omament or good ornament” and “Go in fear of abstraction.” Beyond attention to the image, Pound discusses the merits of translation, the rigors of free verse, and poets of the past. The essay is bracing and energetic; after all: “The mastery of any art is the work of. lifetime.”| 6. In his “Self- Reliance” which two qualities does Emerson refer to as “the Chancellors of God"? (A) Truth (B) Cause (©)Spirit (D) Effect Choose the correct answer from the options given below: (1)(A) and (B) only (2)(A) and (C) only 3) B)and C) only (4) (B)and () only Answer: (4) [“Self-Reliance” is an essay by Ralph Waldo Emerson, first published in the first volume of his collected Essays (1841). Emerson's doctrine of self-sufficiency and self-reliance arose naturally from his view that the individual need only look inward for the spiritual guidance that was previously the province of the established churches. In the essay “Emerson argues that “Cause and effect” are “the chancellors of God”. He also claims that “Cause and effect” ate very opposites of fortune, luck, or chance. By self-reliance, man is able to tum his wheel of fortune, according to his own wish. Understanding the rules of “Cause and efféct ",a self-reliant individual applies his will wisely to achieve desired effects. To put it another way, through the wisdom of self-reliance, people become masters of their own destinies. Just as God is said to have designed order out of chaos, so too can men.] NTA UGC NET 2020 (Evening Shift, Paper-II) 33 7. Which of the following captures accurately the view of Frankfart School of Criticism Theory? (1) The culture industries in still in their mass audiences a capacity to question and transform (2) The culture industries engender passivity and conformity among their mass audiences (3) Power and culture are two distinet modes of social articulation, separate from each other (4) The analysis of culture should be divorced from politics and power relations Answer: (2) [Frankfurt School was founded in 1923, and attracted a diverse faculty of mostly Jewish, left- \wing intellectuals, committed to the interdisciplinary study of society, grounding generalized philosophical thought in concrete sociological research. Those associated with the Frankfurt School include the philosophers Max Horkheimer, Theodor Adorno, Herbert Marcuse and Walter Benjamin, ‘The body of thought that emerged from the Frankfurt School was called ‘Critical Theory’, as distinct from the more generalized modern sense that is common in literary and cultural studies. Horkheimer first used the term inhis paper “Traditional and Critical Theory’ (1937), in which he detailed the Frankfurt School's agenda. Horkheimer considered ‘traditional theory’ to be complicit with capitalism, whereas ‘Critical Theory’ would specifically aim to critique capitalist social relations. Frankfurt Schoo! thinkers used the term ‘advanced industrial society” to describe capitalist and fascist societies, which tended towards both centralization and standardization of culture. In general, they championed esoteric, avant-garde or ‘modernist’ forms of art, which they believed to express a critique of the dominant culture.) 8. Arrange the following plays in their chronological order: (A) The Tempest (B) All For Love (©) Folpone (D) The School for Scandal Choose the correct answer from the options given below: MA), ©,B8),O) 2) ©) (A),B),O) 3) (©), B),(A),O) (4)(A), ©), (B),© Answer: (2) [Volpone is a comedy play by Ben Jonson first produced in 1605-1606. The Tempest isa play by William Shakespeare, probably written in 1610-1611. All for Love is a 1677 heroic drama by John Dryden. 7The School for Scandal (1777) is a comedy of manners written by R. B. Sheridan,] 9. Which of the following are non- fictional works by Peter Ackroyd? (A) Escape from Earth (B) The Great Fire of London (©) The English Ghost (D) English Music Choose the correct answer from the options given below: (1) (A) and (B) only (2)(A) and (C) only (3)(B) and (C) only (4) (B) and (D) only Answer: (2) [Peter Ackroyd is an English biographer, novelist and critic. The Great Fire of London (1982) and English Music (1992) are his best known novels. Escape From Earth (2004) and The English Ghost (2010) are his non- fictional works.} List-I List-I1 Author ‘Autobiography/ Memoir. (@ Pablo Neruda (® Under My Skin (b) Graham Green (ii) Speak, Memory (© Doris Lessing (iii) Memoirs (@Viadimir Nabakov Gi) Sort of Life Q)@PLi), (vi), (i G) @-ii), (6)-Gv), (Gi), @)-G) 4) @MLii), Ov), (CG). (Gi) Answer: (4) [Under My Skin (1994) was the first volume of Doris Lessing’s autobiography, covering the period of her life from birth in 1919 to 1949, The second volume Walking in the Shade appeared in 1997: Speak, Memory (1951) is an autobiographical memoir by Vladimir Nabokov. The classic and deeply moving Memoir by Pablo Neruda was published in 1974. A Sort of Life (1971) is the first volume of autobiography by British novelist Graham Greene.] 11, Who among the following theorists defines novel as “a phenomenon multiform in style and variform in speech and voice? (DEM. Forster (2) Henry James (3) Mikhail Bakhtin (4) Eric Auerbach Answer: (3) [In his book Discourse in the Novel, Mikhail Bakhtin introduces his idea of heteroglossia, based on “extralinguistic” features common across languages, like perspective, evaluation, and ideology. The focus of this essay is the insistence that literary study must neither be “formal” nor “ideological,” but that form and content are unified in discourse. The fixation on style, cut off from the sociality of discourse, is flat and abstract and the two must be put in conversation. “The novel as a whole is a phenomenon multiform in style and variform in speech and voice”. Its “structured artistic system” is made up of direct narration, stylized narration, stylized everyday forms like the letter or diary, other literary but extra-artistic forms like scientific or journalistic texts, and stylized individual speech of characters. They form together “a higher stylistic unity of the work as a whole, a unity that cannot be identified with any single one of the unities subordinated to it”.] 12. Which two of the following novels are parts of Paul Auster’s New York Trilogy? (A) The Book of Mlusions (B) Ghosts (©) The Locked Room (D) Winter Journat Choose the correct answer from the options given below: (1) (A)and (B) only (2)(A) and (©) only (3)(B)and (©) only (4) (B) and (D) only Answer: (3) [The remarkable, acclaimed series of interconnected detective novels, The New York Trilogy is a series of novels by Paul Auster. It was published sequentially as City of Glass (1985), Ghosts (1986) and The Locked Room (1986),] NTA UGC NET 2020 (Evening Shift, Paper-II) 35 13. Arrange the following in the chronological order of publication (A) Modern English Usage (B) Proposal for Perfecting the English Language (C) Usage and Abusage (D) An American Dictionary of English Usage Choose the correct answer from the options given below: (1) ©),B),©,(A) 2)B), ©), (D),(A) G)B),),(A),© (4) ),©,(A),B) Answer: (3) [Thomas Cooke's Proposal for Perfecting the English Language was published in 1742. An American Dictionary of English Usage was published in 1828 by the American lexicographer Noah Webster. Modern English Usage, by Henry Watson Fowler was published in 1926. It is a style guide to British English usage, pronunciation, and writing, Eric Partridge’s Usage and Abusage was published in 1942.] 14. Who wrote the essay “Naipaul’s India and Mine”(1984) as a reply to V.S. Naipaul's “An Area ofDarkness”? (2)A.K. Ramanujan (2) Nissim Ezekiel (3) Nayantara Sahgal (4) Mahesh Dattani Answer: (2) [V. S. Naipaul’s travelogue An Area of Darkness (1964) portrayed a poor and filthy India. It was highlighted because of its negative portrayal of India and Indians. Naipaul may have been suggesting that for him, India was and remained an area of darkness. He provoked sharp retorts in India and the book was banned. It was categorized alongside Mother India in which Katherine Mayo had attacked India’s society, culture and religion, Poet Nissim Ezekiel issued a famous rejoinder “Naipaul’s India and Mine”, criticizing the writer for his description of the grossness and squalor of Indian life, the routine ritualism, the lip- service to high ideals, the petrified and distorted sense of cleanliness and a thousand other things.] 15. Given below are two statements: One is labeled as Assertion A and the other is labeled as ReasonR Assertion (A) : English today is not only the language we teach but also the subject that enables itslearners to become subtle and tough minded readers Reason (R): Students are encouraged to think and analyses the historical and ontological status of the texts they read and how best to read them, In the light of the above statements choose the most appropriate answer from the options given below: (1) Both (A) and (R) are correct and (R) is the correct explanation of (A) (2) Both (A) and (R) are correct but (R) is NOT the correct explanation of (A) (3) (A) is correct but (R) is not correct (4) (A) is not correct but (R) is correct Answer: (1) [Both statements are correct and logically connected.] 16. In which Act of William Congreve’s The Way of the World does the Proviso scene between MirabellandMillamant take place? ()Actl (2) Act IL (3) Act TIL ()ActIV Answer: (4) [The proviso scene in The Way of the World (1700) is generally considered the finest in Restoration comedy. It takes place in Act IV.In the scene Mirabell and Millament meeting together to arrange an agreement for their marriage. The scene is a pure comedy with brilliant display of wit by both of them, but, above all, provides instructions which have serious dimensions in the context of the society. Here, Congreve seems to come to realise the importance for providing an ideal pair of man and woman, ideal in the sense that the pair could be taken for models in the life-style of the period.] 36 BRUSHUPYOUR ENGLISHLITERATURE 17. Which agency among the following was of the view that “use of English, — divides the people into two nations,the few who govern and the many who are governed”? (1) The Kunzru Committee (1955) (2) The Education Commission (1948) (3) The Education commission (1964-66) (4) The working Group (UGC) on Regional Languages (1978) Answer: (2) [Education Commission in 1948 recommended that, “English has becomes so much a part of our national habit that a plunge into an altogether different system seems attended with unusual risks. It appears to us, however, that the plunge is inevitable. English cannot continue to occupy the place of state language as in the past. Use of English as such divides the people into two nations, the few who govern and the many who are governed, the one unable to talk the language of the other and mutually uncomprehending. This is a negation of democracy.”] 18. Which of the following groups of words correctly states the stages of communication as envisioned by Stuart Hall in his essay “Encoding, Decoding”? (1) Production, transference, circulation, contact, reproduction (2) Production, circulation, realization, consumption. Reproduction (3) Production, circulation, distribution, consumption, reproduction (4) Production, dissemination. transference, consumption, reproduction Answer: (3) [Most widely circulated and debated of all Hall’s papers, ‘Encoding/decoding’had a major impact on the direction of cultural studies in the 1970s and 1980s and its central terms remain keywords in the field “Encoding/decoding’ arises primarily from Hall’s reservations about the theories of communication underpinning ‘mass communications research. According to ‘encoding/decoding’ model, any communication goes through stages from the time: a message is encoded (production) and transmitted (circulation) to when it is decoded (consumption) and reacted upon (reproduction).] 19. Which two poems in the following list are Odes Written in the Horatian manner? (A) Ben Jonson, “To the Immortal Memory and Friendship of that Noble Pair, Sir Ludy.: Cary and Sir H.Morison” (B) Andrew Marwell, “Upon Cromwell's Retum from Ireland” (©)Alexander Pope, “Ode on Solitude” (D) Alfied Tennyson, “Ode on the Death of the Duke of Wellington” ‘Choose the correct answer from the options given below: (1) (A) and (B) Only (2) (B) and (C) Only (3)(C) and (D) Only (4) (A) and (D) Only Answer: (2) [The Horatian odewas originally modeled on the matter, tone, and form of the odes of the Roman Horace. In contrast to the passion, visionary boldness, and formal language of Pindar’s odes, many Horatian odes are calm, meditative, and colloquial; they are also usually homostrophic(that is, written in a single repeated stanza form), and shorter than the Pindaric ode. Examples are Marvell’s “An Horatian Ode upon Cromwell's Return from Ireland” (1650), Alexander Pope, “Ode on Solitude” and Keats’ ode “To Autumn” (1820).] 20, Mr. Pumblechook is a character in: (1) Little Dorret (2) Nicholas Nickleby (3) Hard Times (4) Great Expectations Answer: (4) [Charles Dickens is justly famous for his vivid secondary characters and for their names, which are ofien deliciously absurd, like that of Mr. Pumblechook. Mr. Pumblechook appears in Great Expectations. He is introduced to us as ‘a large hard-breathing middle-aged slow man’ (Chapter 4). He’s the uncle of gentle Joe Gargery, Pip’s brother-in-law and father figure. Since Mr. Pumblechook has some social clout, he’s ‘appropriated’ by Pip’s sister, who shares Mr. Pumblechook’s aggressive self-righteousness.] NTA UGC NET 2020 (Evening Shift, Paper-II) 37 21. While assembling a working bibliography which two of the following reference sources beparticularly useful to a literary researcher? (A)MLA International Bibliography (B) New Princeton Encyclopaedia of Poetry and Poetics (C) Library of Congress Catalogue (D) Reader’s Guide to Periodical Literature Choose the correct answer from the options given below: (1)(A)and (B) only (2)(B)and (C)only (3) (A) and ()only (4) (©and (D) only Answer: (3) [MLA International Bibliography is an index of scholarship on all aspects of modern literature and languages. It indexes bibliographies, dissertations, journal articles, monographs, proceedings, series, scholarly websites, etc. It includes over 2.8 million citations, and over 66,000 are added each year. The Reader's Guide to Periodical Literature is an index of 300 general-interest periodicals in the English language.] 22.Who among the following held that “the people of Hindustan” are “a race of men lamentably degenerate and base, retaining but a feeble sense of obligation...”? (1) Charles Wilkins (2) Thomas Macaulay (3) Charles Grant (4)David Hare Answer: (3) [During British rule the Indians have been excluded from every honour, dignity, or office, which the lowest Englishmen could be prevailed to accept. Britishers believed that an administration based on British ideas, institutions, and practices could be firmly established only by English personnel. And, then, they did not trust the ability and integrity of the Indians. For example, Charles Grant, Chairman of the Court of Directors, condemned the people of India as “a race of men lamentably degenerate and base; retaining but a feeble sense of ‘moral obligation; and sunk in misery by their vices”. Similarly, Comwallis believed that “Every native of Hindustan is comupt.”] 23. In which two of the following plays does the blind seer, Tiresias, appear? (A) Oedipus the King (B) Agamemnon (C)Antigone (D) Oedipus at Colonus Choose the correct answer from the options given below: (1) (A) and (B) Only (2) (A)and (C) Only (3) (B) and (C) Only (4) (C)and (D) Only Answer: (2) (Tiresias, in Greek mythology, a blind Theban seer, the son of one of Athena’s favourites, the nymph Chariclo, He is a participant in several well-known legends. Tiresias appears in all of the Greek tragedies that take place in Thebes—Oedipus Rex, Antigone, and The Phoenician Women—as well as in Euripides’ The Bacchae and in Homer's The Odyssey’] 24. Arrange the following 19Century magazines in the chronological order of their publication: (A) The London Magazine (B) Quarterly Review (C) The Spectator (D) Edinburgh Review Choose the correct answer from the options given below: 38 BRUSHUPYOUR ENGLISHLITERATURE, MA).O),©.B) (2)(B),(A),(D), (©) @)@),B8),(4),©) ©), ©), (B),(A) Answer: (*) [The Spectator was a daily publication founded by Joseph Addison and Richard Steele in England, lasting ftom 1711 to 1712. The London Magazine is England’s oldest literary periodical, with a history from 1732 to till today.The Edinburgh Review was a Scottish magazine that was published from 1802 to 1929. The Quarterly Review was a literary and political periodical founded in March 1809 by the well known London publishing house John Murray. It ceased publication in 1967.] 25. In Harold Pinter’s The Birthday Party, who suggest the idea of having a birthday party? (1) Meg, (2) Goldberg, @)Lulu (4) McCann Answer: (2) [The Birthday Party (1859) is a play by Harold Pinter. The action takes place entirely in a shabby rooming house where Stanley, a lazy young boarder, is shaken out of his false sense of security by the arrival of two mysterious men who proceed to “punish” him for crimes that remain unrevealed. A birthday party is suggested by Goldberg] 26. Which two of the following words are borrowed into English from Czech? (A) pistol (B) robot (©) sauna (D) coach ‘Choose the correct answer from the options given below : (1) (A)and (B) Only (2)(A)and (C) Only (3) (B) and (C) Only (4)(A)and (D) Only Answer: (1) [The word ‘pistol’ is taken from the obsolete French word ‘pistole’, which is taken from German Pistole, which again is taken from the Czech word ‘pis’tala’ of which the original meaning was ‘whistle’, hence ‘a firearm’ by the resemblance in shape. The word robot is taken from Czech, from robota meaning ‘forced labour’ ] 27. Which two of the following periodicals are devoted to feminist theoretical discussion? (A) Spectrum (B) Signs (©) Chrysalis () Transition ‘Choose the correct answer from the options given below: (1)(B)and (C) Only (2)(A) and (©) Only (3)(B)and (D) Only (4)(A) and (D) Only Answer: (1) [Recognized as the leading international journal in women’s and gender studies, Signs is at the forefront of new directions in feminist scholarship. Signs publishes path breaking articles, review essays, comparative perspectives, and retrospectives of interdisciplinary interest addressing gender, race, culture, class, nation, and sexuality.Signsis currently edited by Suzanna Danuta Walters, and Carla Kaplan. Chrysalis: A Magazine of Women’s Culture was a feminist publication produced from 1977 to 1980.] 28. Who among the following refutes Plato’s charge that poets are liars, by arguing that the poet"nothing affirms, and therefore never lieth”? (1) John Dryden (2) Philip Sidney (3) George Puttenham (4) Richard Hooker Answer: (2) [Im his attempt to refute Plato’s critique of the poets in the second book of The Republie, where they were accused of telling lies, Sir Philip Sidney stated in his well-known Defense of Poesie: “Now for the poet, NTA UGC NET 2020 (Evening Shift, Paper-II he nothing affirmeth, and therefore never lieth. For, as I take it, to lie is to affirm that to be true which is false; so as the other artists, and especially the historian, affirming many things, can, in the cloudy knowledge of mankind, hardly escape from many lies. But the poet, as I said before, never affirmeth...”] 29. Arrange the following novels in the chronological order of their publication: (A) The White Tiger (B)A Tiger for Malgudi (©)A Suitable Boy (D) Heat and Dust Choose the correct answer from the options given below: (1) @),B),(©), (A) (2)B),(D),(©),(A) (3)(B).(©),(A),D) (4)(B),(C),D), (A) Answer: (1) [Booker Prize winning novel Heat and Dust (1975) is written by Ruth PrawerJhabvala. R.K. Narayan's A Tiger for Malgudi is a novel published in 1983. Iti told by a tiger in the first person, Vikram Seth’s novel 4 Suitable Boy was published in 1993. Aravind Adiga’s Booker awardee novel White Tiger was published in 2008. ] 30. Who is the author of The Complete Plain Words? (1) Samuel Jhonson (2) Daniel Jones (3) Emest Gowers (4) Michael Everson Answer: (3) [The Complete Plain Words is a style guide written by Emest Gowers, published in 1954. It comprises expanded and revised versions of two pamphlets Plain Words (1948) and ABC of Plain Words (1951).] 31. Match List- I with List IT List-I List-I Novel Character (a) Barnaby Rudge (i) Miss La Creevy (b) Little Dorrit (ii) Miss Dolly (c) Nicholas Nickleby (iii) Mrs. Boffin (@) Our Mutual Friend (iv)Mrs. Flintwinch (1) @{i), 0), (Gi), @-Civ) 2) @Hiii), )-Gi), (Civ), @-) 3) (@)-Ci), (0)4iv), (4), (@)(ii) A) @Hiv), (6), (©-Gii), @)-Gi) Answer: (3) [Miss La Creevy is a character in Charles Dickens’s novel Nicholas Nickleby (1838). Miss Dolly isa character in Charles Dickens’s novel Barnaby Rudge (1839). Mrs. Boffin is a character in Charles Dickens's novel Our Mutual Friend (1865).Mrs. Flintwinch is a character in Charles Dickens’s novel Little Dorrit (1857). 32. Which two of the following edited the defaming work of third wave of feminism, This Bridge Called My Back: Writing by Radical Women of Color? (A) AudreLorde (B) Barbara Smith (C) Gloria Anzaldua (D) Cherrie Moraga Choose the correct answer from the options given below: (1) (A) and (B) Only (2) (C) and (D) Only (3) (A)and (©) Only (3) (B) and (D) Only Answer: (2) [This Bridge Called My Back: Writings by Radical Women of Color isa feminist anthology edited 40 BRUSHUPYOURENGLISHLITERAI by Cherrie Moraga and Gloria Anzaldua. It was first published in 1981. It is an anthology of essays, theory, fiction, poetry, and the fusion of all four written by radical women of color.] 33. Who is the author of the short play, The Dark Lady of the Sonnets? (1) Ben Jonson (2) George Bernard Shaw (3) Oscar Wilde (4) Oliver Goldsmith Answer: (2) [George Bernard Shaw's The Dark Lady of the Sonnets (1910) is a one act play that reimagines William Shakespeare’s and Queen Elizabeth’s first meeting. Written as an appeal to create a National Theatre in England, the short play has William going to Whitehall palace to meet the Dark Lady to whom he addresses his sonnets of love, only to fall for the Queen herself as they randomly meet! As Will juggles both women’s tempers and desires, he finds himself inspired to write new material and appeal for a theatre where bold tragedies will be taken seriously.) 34. Arrange in the chronolo; (A) The Unfinished Man (B) Gitanjali (©sejuri (D) TheSceptred Flute ‘Choose the correct answer from the options given below : (1) B), (A), D), (©) (2) ©), (B),(©), (A) @)®),),(A),.© 4) ®),O),©.(A) Answer: (3) [Gitanjali is a collection of poems Rabindranath Tagore. It was originally published in 1910. The Sceptred Flute Songs of India contains the complete poetical works Sarojini Naidu. It was published in 1943, Nissim Ezekiel’s The Unfinished Man is a poetry collection published in 1959. Jejuri is a series of poems written by ArunKolatkar published in 1974.] 35. Given below are two statements: Statement I : The Education Commission (1964-66) recommended the removal of English as a medium of instruction at the college level. Statement II : English is still largely the language of administration and jurisprudence in India. In the light of the above statements, choose the correct answer from the options given below: (1) Both Statement I and Statement Il are true (2) Both Statement I and Statement II are false (3) Statement I is correct but Statement II is false (4) Statement I is incorrect but Statement IT is true Answer: (4) [Education Commission (1964-66) recommended adopting a three-language formula at state levels. It intended to promote a language of the Southern States in Hindi speaking states. It intended to promote Hindi, English, and a regional language in non-Hindi speaking states.] 36. Metaphor differs from simile in that (1) Acomparison in metaphor is usually explicit whereas in simile itis implicit (2) A comparison in metaphor is usually implicit whereas in simile itis explicit (3) Neither metaphor nor simile is rooted in comparison (4) Simile involves superimposition while metaphor involves comparison Answer: (2) [Metaphor, the most important and widespread figure of speech, in which one thing, idea, or action is referred to by a word or expression normally denoting another thing, idea, or action, so as to suggest some common quality shared by the two. In metaphor, this resemblance is assumed as an imaginary identity rather than directly stated as a comparison. A comparison in metaphor is usually implicit whereas in simile it is explicit.] 37. Arrange the following in the chronological order of their publication: (A) Past and Present (B) Leviathan NTA UGC NET 2020 (Evening Shift, Paper-I) 41 (©) Unto This Last (D) The Life of Samuel Johnson Choose the correct answer from the options given below: ()®).©),),© QO.O).A).8) QB.A.OM )©,@),0),B) Answer: (1) [Leviathan is a book written by Thomas Hobbes and published in 1651. The Life of Samuel Johnson (1791) by James Boswell is a biography of Dr. Samuel Johnson.Past and Present is a book by Thomas Carlyle. It was published in 1843.Unto This Last is a book on economy by John Ruskin, first published in1860.] 38. Language allows us to talk about the things and events not present in immediate environment. Which of the following terms describes this property of language? (1) Arbitrariness (2) Displacement (3) Productivity (4) Discretenes Answer: (2) [Displacement: In linguistics, a characteristic of language that allows users to talk about things and events other than those occurring in the here and now. Displacement is one of the distinct properties of human language. Its significance as one of the 16 “design features of language” was noted by American linguist Charles Hockett in 1960.] 39, How does Christ respond to the Grand Inquisitor’s accusations in Brothers Karamazov? (1) He kneels before the Grand Inquisitor (2) He kisses the Grand Inquisitor on his lips (3) He begins to weep in remorse (4) He says, “Mea culpa, mia culpa, mia maxima culpa” Answer: (2) [The Grand Inquisitor is the most famous chapter in Fyodor Dostoyevski's, The Brothers Karamazov (1880). In this chapter, The Grand Inquisitor sings a long, painful soliloquy. Jesus remains silent, throughout his soliloquy. The story ends when Jesus, instead of answering, simply kisses the Grand Inquisitor on his “bloodless, aged lips”. On this, the Inquisitor releases Jesus, telling him never to come back. Still silent, Jesus walks away into “the dark alleys of the city.”] 40. Arrange the following critical works in the chronological order of publication: (A) “Preface to Lyrical Ballads” (B)A Defence of Rhyme (© “Life of Cowley” (D) “Frontier of Criticism” Choose the correct answer from the options given below: (1)(A),(C),(B), D) (2)(B), (A), ©, ) 3) B), (©), (A),D) (4)(©,(A), ©), (B) Answer: (3) [Samuel Daniel's The Defense of Rhyme was published in} 503, Samuel Johnson’s Life of Cowley from Samuel Johnson’s Lives of the Poets series, published between 1779 and 1781. The Preface to the Lyrical Ballads (1800) is an essay by William Wordsworth, for the second edition of the poetry collection Lyrical Ballads, and then greatly expanded in the third edition of 1802.T. S. Eliot’s “The Frontier of Criticism” was published in 1956] 41, Which two of the following were published in the year 1859? (A) On the Origin of Species (B)A Tale of Two Cities (©) Alice in Wonderland

You might also like